PPS COMPILED SAMPLEX [PART 4 OF 5] - 652 items total with Rationale Flashcards

1
Q

“Which should be given attention after birth?

a. Cephalhematoma
b. Caput succedaneum
c. Tongue thrusting”

A

A

“Nelson 21st p869
Caput succedaneum, causd by scalp pressure from the uterus, cervix, or pelvis, appears as a circular boggy area of edema with indistinct borders and often with overlying ecchymosis.

A cephalhematoma presents as a well-circumscribed fluid filled mass that does not cross suture lines. Unlike caput succedaneum, cephalhematoma is often not present at delivery but develops over the 1st few hr of life.

Both cephalhematoma and caput succedaeneum must be distinguished from a subgaleal hemorrhage, which is not restricted bby the boundaries of the sutures and therefore is larger and more diffuse. Subgaleal hemorrhage requires prompt recognition because extensive bleeding may result in hypovolemic shock, with estimated mortality up to 20% “

How well did you know this?
1
Not at all
2
3
4
5
Perfectly
2
Q

Cause of symmetric facial palsy

A

Hypoplasia of 7th nerve nucleus: Mobius syndrome

How well did you know this?
1
Not at all
2
3
4
5
Perfectly
3
Q

“Newborn with asphyxia, cyanotic at birth. Responds intermittently to 100% 02 via hood. CXR is normal. What to consider?

a. Congenital heart disease
b. PPHN
c. Respiratrory distress syndrome”

A

B

“Nelson 21st p2395
The hyperoxia test is one method of distinguishing cyanotic CHD from pulmonary disease. Neonates with cyanotic CHD are usually unable to significantly raise their arterial partial pressure of O2 (PaO2) during administration of 100% O2. This test is usually performe dusing a hood rather tha nasal cannula or FM, to best guarantee delivery of almost 100% O2 to the infant.

If the PaO2 rises above 150mmHg during 100% O2 administration, an intracardiac right-to-left shunt can usually be excluded. In patients with pulmonary disease, PaO2 usually increases significantly with 100% O2 as ventilation-perfusion inequalities are overcome.

Hyperoxia in many heart conditions is profound and constant, whereas in respiratory disorders and PPHN, PaO2 often varies with time or changes in ventilator management. Hyperventilation may improve the hypoxia in neonates with PPHN and only occasionally in those with cyanotic CHD. “

How well did you know this?
1
Not at all
2
3
4
5
Perfectly
4
Q

“Period of teratogenesis?

a. 1-14 days
b. 14 days to 9 weeks
c. 9 weeks”

A

B

“Nelson 21st p886
Overall, only 10% of anomalies are caused by recognizable teratogens. The time of exposurue that is most likely to cause injury is usually during organogenesis at <60 days of gestation. “

How well did you know this?
1
Not at all
2
3
4
5
Perfectly
5
Q

“Chorioamionitis greatest risk factor with this etiology:

a. E. Coli
b. Listeria
c. Group B strep”

A

C

“Nelson 21st p 1011
Chorioamnionitis has been historically used to refer to microbial invasion of the amniotic fluid, often as a result of prolonged rupture of the chorioamniotic membrane for >18hr. Prematurity (<37wk) is associated with a greater risk of early-onset sepsis, especially with group B streptococcus “

How well did you know this?
1
Not at all
2
3
4
5
Perfectly
6
Q

“Skin to skin contact benefits except:

a. Maternal flora
b. Warmth
c. Facilitates breastfeeding
d. Prevents anemia”

A

D

How well did you know this?
1
Not at all
2
3
4
5
Perfectly
7
Q
"Which is true of cord clamping 
A. 2 cm above base
B. Milking
C. Both of the above
D. None of the above"
A

A

How well did you know this?
1
Not at all
2
3
4
5
Perfectly
8
Q
"True of caput except 
A. Crosses midline
B. Subperiosteal bleeding
C. With edema
D. Resorption at 3-4 days"
A

B

”"”Nelson 21st p869
Caput succedaneum, causd by scalp pressure from the uterus, cervix, or pelvis, appears as a circular boggy area of edema with indistinct borders and often with overlying ecchymosis.

A cephalhematoma presents as a well-circumscribed fluid filled mass that does not cross suture lines. Unlike caput succedaneum, cephalhematoma is often not present at delivery but develops over the 1st few hr of life.

Both cephalhematoma and caput succedaeneum must be distinguished from a subgaleal hemorrhage, which is not restricted bby the boundaries of the sutures and therefore is larger and more diffuse. Subgaleal hemorrhage requires prompt recognition because extensive bleeding may result in hypovolemic shock, with estimated mortality up to 20% “””

How well did you know this?
1
Not at all
2
3
4
5
Perfectly
9
Q
"White pearly papules with erythematous base. 
A. Milia
B. Erythema toxicum
C. Pustular melanosis
D. None of the above"
A

B

“Neonatal Skin Lesions
1. Sebaceous hyperplasia - minute profuse yellow-white papules in forehead, nose, upper lip, cheeks

  1. Milia - superficial epidermal inclusion cysts with laminated keratinized materia, firm pearly opalescent white cyst 1-2mm diamter on face and gingivae and in the middle of palate (Epstein pearls)
  2. Cutis marmorata - evanescent lacy reticulated red and blue cutaneous vascular pattern over most of the body surface when infant is cold
  3. Harlequin color change - autonomic vascular regulation instability, longitudinal half pale, half red
  4. Nevus simplex (salmon patch) - small pale pink ill defined vascular macule on glabella eyelids, upper lip, nuchal area
  5. Dermal melanocytosis (mongolian spots) - blue or slate gray macular lesions with variably defined margins most common on sacral area
  6. Erythema toxicum - benign self limited evanescent eruption; firm yellow-white 1-2mm papules or pustules with surrounding erythematous flare; eosinophilic infiltrates
  7. Neonatal pustular melanosis - transient benign self-limited dermatosis; evanescent superficial pustules, ruptured pustules with colarette of fine scale, hyperpigmented macules; polymorphonuclear infiltrates

(Nelson 21st 3453-3455)”

How well did you know this?
1
Not at all
2
3
4
5
Perfectly
10
Q
"Most characteristic of shaken baby
A. Retinal hemorrhages 
B. Clavicular fracture
C. Limb fracture 
D. None of the above"
A

A

“Abusive head trauma / shaken baby syndrome - poor neck muscle tone and relatively large heads of infants make them vulnerable to acceleration-deceleration forces from shaking.

Features of AHT (sensitive especially when occurring together)

  1. Subdural hematoma
  2. Retinal hemorrhages - multiple, involve >1 layer of retina and extend to periphery; traumatic retinoschisis
  3. Diffuse actional injury

(Nelson 21st p104)”

How well did you know this?
1
Not at all
2
3
4
5
Perfectly
11
Q
"Lacy reticulated skin changes of a neonate exposed to cold surroundings.
A. Harlequin color change
B. Erythema toxicum
C. Neonatal sebaceous hyperplasia 
D. Cutis marmorata"
A

D

“Neonatal Skin Lesions
1. Sebaceous hyperplasia - minute profuse yellow-white papules in forehead, nose, upper lip, cheeks

  1. Milia - superficial epidermal inclusion cysts with laminated keratinized materia, firm pearly opalescent white cyst 1-2mm diamter on face and gingivae and in the middle of palate (Epstein pearls)
  2. Cutis marmorata - evanescent lacy reticulated red and blue cutaneous vascular pattern over most of the body surface when infant is cold
  3. Harlequin color change - autonomic vascular regulation instability, longitudinal half pale, half red
  4. Nevus simplex (salmon patch) - small pale pink ill defined vascular macule on glabella eyelids, upper lip, nuchal area
  5. Dermal melanocytosis (mongolian spots) - blue or slate gray macular lesions with variably defined margins most common on sacral area
  6. Erythema toxicum - benign self limited evanescent eruption; firm yellow-white 1-2mm papules or pustules with surrounding erythematous flare; eosinophilic infiltrates
  7. Neonatal pustular melanosis - transient benign self-limited dermatosis; evanescent superficial pustules, ruptured pustules with colarette of fine scale, hyperpigmented macules; polymorphonuclear infiltrates

(Nelson 21st 3453-3455)”

How well did you know this?
1
Not at all
2
3
4
5
Perfectly
12
Q
"Breastmilk jaundice true of the following 
A. Starts within the 3rd day of life
B. Starts after 7 days
C. Caused by dehydraton 
D. Treatment is DVET"
A

B

“Breastfeeding jaundice

    - 3rd-4th DOL
    - Inadequate supply of breastmilk leading to increased enterohepatic circulation
    - Tx: Increase breastfeeding to 8-10x/day 

Breast milk jaundice

    - 1st-2nd week of life
    - Glucoronidase in breast milk increases enterohepatic circulation
    - Tx: Increase breastfeeding frequency, phototherapy"
How well did you know this?
1
Not at all
2
3
4
5
Perfectly
13
Q
"Most common symptom of neonatal tetanus
A. Progressive poor suck
B. Fever
C. Ophistotonos 
D. Paralysis"
A

A

“Neonatal tetanus, the infantile form of generalized tetanus, typically manifests within 3-12 days of birth. It presents as progressive difficulty in feeding (sucking and swallowing), associated hunger, and crying.

Paralysis or diminished movement, stiffness and rigidity to the touch, and spasms, with or without opisthotonos, are characteristic. The umbilical stump, which is typically the portal of entry for the microorganism, may retain remnants of dirt, dung, clotted blood, or serum, or it may appear relatively benign.

(Nelson 21st p1550)”

How well did you know this?
1
Not at all
2
3
4
5
Perfectly
14
Q

“Treatment for primary apnea of prematurity. Which is true.
A. Gentle tactile stimulation is enough for primary apnea
B. Theophylline is more effective than caffeine
C. High flow NC is more preferred than CPAP
D. All of the above”

A

A

“Management of apnea of prematurity

  1. Gentle tactile stimulation or provision of flow and/or supplemental oxygen by nasal cannula is often adequate therapy for mild and intermittent episodes
  2. Nasal CPAP and heated humidified high flow nasal cannula are appropriate therapies. nCPAP may be preferred in extremely preterm infants
  3. Recurrent or persistent apnea is treated with methylxanthines (increase respiratory drive by lowering the threshold of response to hypercapnia)
  4. Caffeine and theophylline are similarly effective, but caffeine is preferred due to longer half life and less side effects (tachycardia, feeding intolerance)
  5. Caffeine PO or IV with 20mkdose loading then maintained at 5-10mkday

(Nelson 21st p931)”

How well did you know this?
1
Not at all
2
3
4
5
Perfectly
15
Q
"When is gender known via fetal UTZ 
A. 12 weeks
B. 14 weeks
C. 18 weeks
D. 20 weeks"
A

A

How well did you know this?
1
Not at all
2
3
4
5
Perfectly
16
Q

“For a child who was delivered non institutionally, until what month should hearing screen be ideally done?

a. 1 month
b. 2 months
c. 3 months
d. 4 months”

A

C

“Prev Ped 2018 p9
All infants born in hospitals in the Philippines shall be made to undergo newborn hearing loss screening before discharge, unless the parents or legal guardians of the newborn object to the screening. Infants who are not born in hospitals should be screening within 3 months of birth.

In the event of a positive newborn hearing loss screening result, the newborn shall undergo audiologic diagnostic evaluation in a timely manner to allow appropriate followup, recall, and referral for intervention before the age of 6 months. “

How well did you know this?
1
Not at all
2
3
4
5
Perfectly
17
Q

“A healthy full term infant was delivered via spontaneous vaginal de livery and with unremarkable course. He was discharged within 24 hours. When’s the latest time that the baby must be seen at the clinic for follow up?

a. 36 hrs
b. 48 hrs
c. 72 hrs
d. 96 hrs”

A

B

“Prev Ped 2018 p15 Appendix 2. Discharge and followup of healthy term newborns

The Philippine Society of Newborn Medicine lists the following minimum criteria for discharging newborns within 48hrs

  1. Uncomplicated antepartum, intrapartum, and postpartum courses for both mother and newborn
  2. Vaginal delivery, singleton, completed 37 weeks AGA
  3. Normal and stable VS during the preceding 12hrs
  4. Has urinated and passed at least one stool
  5. Has documented proper latch, milk transfer, swallowing, infant satiety and absence of nipple discomfort
  6. Normal PE
  7. No evidence of significant jaundice in first 24hrs of life
  8. Educability and ability of parents to care for their child (recognize signs of illness, care of the umbilical cord/skin/genitalia, maternal confidence in feeding her infant and parents’ understanding of the importance of ffup visit or emergency consultation
  9. Must followup within the next 48hrs”
How well did you know this?
1
Not at all
2
3
4
5
Perfectly
18
Q

“A baby left exposed and hypothermic could develop the following complications except:

a. Hypoglycemia
b. Hypoxia
c. Metabolic acidosis
d. Renal retention and will not urinate”

A

D

“Nelson 21st p913
Cold stress can lead to profound decompensation, including apnea, bradycardia, respiratory distress, hypoglycemia, and poor feeding. For this reason, it is paramount for the neonate to maintain normothermia in the delivery room and afterward, especially low birthweight and premature infants. For VLBW infants, a combination of occulisive plasic wrap, radiant warmers, and thermal matresses to maintain normothermmia can be used to reduce cold stress. “

How well did you know this?
1
Not at all
2
3
4
5
Perfectly
19
Q

“Newborn screen can be done on full term healthy baby until:

a. 48 hours
b. 72 hours
c. 96 hours”

A

B

“Prev Ped 2018 p8
Newborn screening should ideally be done immediately after 24 hrs from birth, regardless of gestational age and clinical status.

NSRC Memorandum 2014-028 states that preterm, LBW (<2000g) and sick neonates recieving intensive care may have a sample taken before 24hrs from birth if blood transfusion will be done but otherwise a sample should be taken immediately after 24hrs from birth and a repeat screening should be collected at the 28th day of life. “

How well did you know this?
1
Not at all
2
3
4
5
Perfectly
20
Q

“Which would help in decreasing the incidence of neonatal sepsis?

a. early skin to skin contact
b. timely cord clamping
c. drying and thermoregulation
d. nonseparation of mother and newborn”

A

A

“Fundamentals of Pediatrics vol 1 p85
Skin to skin contact and the principle of nonseparation between mother and newborn has been demonstrated to reduce the incidence of neonatal infection by virtue of the following mechanisms:
1. Colonization of the newborn by maternal skin flora
2. Stimulation of the neonates mucosa-associated lymphoid tissues
3. Less exposure to potentially harmful health care related microorganisms such as those found in neonatal units”

How well did you know this?
1
Not at all
2
3
4
5
Perfectly
21
Q

“Physiologic parameters measured by Apgar score:

a. Color, HR, muscle tone
b. Color, respiration, HR
c. Respiration, muscle tone, reflex response
d. Color, reflex response, HR”

A

B

“Fundamentals of Pediatrics vol 1 p83
The Apgar score measures 3 physiologic parameters (color, respiration, and heart rate) and 2 neurologic parameters (muscle tone and reflex response) “

How well did you know this?
1
Not at all
2
3
4
5
Perfectly
22
Q

“Initial response to hypoxia of neonates

a. Increase in respiration then primary apnea
b. Hypotension & bradycardia
c. Increase in respirations only
d. None of the above”

A

A


Fundamentals of Pediatrics vol 1 p87
The fetus and newborn initially respond to hypoxic insult with rapid breathing followed by primary apnea, which leads to bradycardia without hypotension. “

How well did you know this?
1
Not at all
2
3
4
5
Perfectly
23
Q

“Increases the risk for ROP?

a. < 32 weeks
b. < 1500 grams
c. > 32 weeks and> 1500 grams but with a stormy course
d. All of the above”

A

D

“Prev Ped 2018 p21
Criteria for screening retinopathy of prematurity
1. 32 weeks or younger AOG
2. 1500g or lighter
3. Older than 32 weeks or heavier than 1500g but with stormy medical course at the NICU”

How well did you know this?
1
Not at all
2
3
4
5
Perfectly
24
Q
"A preterm infant is suspected of having NEC. What is the imaging of choice?
A. Ultrasound
B. Plain film of the abdomen
C. CT scan
D. MRI"
A

B

“Nelson 21st p951-952
A very high risk of suspicion in treating preterm at-risk infants is cruical. Plain abdominal radiographs are essential to make a diagnosis of NEC. The finding of pneumatosis intestinalis (air in the bowel wall) confirms the clinical suspicion of NEC and is diagnostic; 50-75% of patients have pneumatosis when treatment is started. Portal venous gas is a sign of severe disease, and pneumoperitoneum indicates a perforation.

Ultrasound with doppler flow assessment may be useful to evaluate for free fluid, abscess and bowel wall thickness, peristalsis, and perfusion. “

How well did you know this?
1
Not at all
2
3
4
5
Perfectly
25
Q

“8 day old baby came in because of jaundice. The baby has good suck and activity and is purely breastfed. What will you do?

a. Stop breastfeeding and shift to formula
b. Continue breastfeeding to increase to at least 10x/day in 24 hours
c. Observe”

A

B

”"”Breastfeeding jaundice

    - 3rd-4th DOL
    - Inadequate supply of breastmilk leading to increased enterohepatic circulation
    - Tx: Increase breastfeeding to 8-10x/day 

Breast milk jaundice

    - 1st-2nd week of life
    - Glucoronidase in breast milk increases enterohepatic circulation
    - Tx: Increase breastfeeding frequency, phototherapy"""
How well did you know this?
1
Not at all
2
3
4
5
Perfectly
26
Q

“A pregnant teenager with only one prenatal check-up gave birth. At 2months into the pregnancy the mother had an episode of rash. The baby was born with a weight of 1.8 kilograms with rashes over the body. Mother claims that she had an ““allergic rash”” during pregnancy . What would you do?
A. Do routine newborn care
B. Isolate the contact for 1 year (baby)”

A

A

Not sure, depends on other sources. Infant seems to have a TORCH infection. Still do routine newborn care, but workup for TORCH.

How well did you know this?
1
Not at all
2
3
4
5
Perfectly
27
Q
"Newborn baby was doing well until at 3 days old, the patient was noted to have increased direct bilirubin, normal coombs and reticulocyte count. Diagnosis?
A. Sepsis
B. Breastfeeding jaundice
C. Breastmilk jaundice
D. Billiary atresia"
A

D

How well did you know this?
1
Not at all
2
3
4
5
Perfectly
28
Q
"The mother contracted varicella 2 days prior to delivery. What would you do?
A. Give varicella vaccine
B, Give immunoglobulin
C, Give Varicella immunoglobulin
D, Give toxoid"
A

C

“Nelson 21st p1715
High titer anti-VZV immune globulin as postexposure prophylaxis is recommended for immunocompromised children, pregnant women, and newborns exposed to varicella: Newborns whose mothers have varicella 5 days before to 2 days after delivery should recieve VariZIG. VariZIG is also indicated for pregnant women and immunocompromised persons without evidence of varicella immunity.”

How well did you know this?
1
Not at all
2
3
4
5
Perfectly
29
Q

“Newborn child with 2 lower central incisor teeth. On PE it does not move on manipulation. What will you do?
A. Do not do anything
B. Pull out the teeth. Risk of aspirations
C. Refer to geneticist”

A

A

“Nelson 21st p1912
Natal or neonatal teeth occasionally result in pain and refusal to feed and can produce maternal discomfort because of abrasion or biting of the nipple during nursing. If the tooth is mobile, there is a danger of detachment, with aspiration of the tooth. Because the tongue lies between the alveolar processes during birth, it can become lacerated (Riga-Fede disease). Decisions regarding extraction of prematurely erupted primary teeth must be made on an individual basis.”

How well did you know this?
1
Not at all
2
3
4
5
Perfectly
30
Q
"What will you do to prevent dental caries in a exclusively breastfed 6 month old infant?
A. Breastfeeding
B. Bottle feeding
C. Complimentary feeding
D. Fluoride supplementation"
A

A

How well did you know this?
1
Not at all
2
3
4
5
Perfectly
31
Q

“What neonatal factor is responsible for predisposition to infection in preterms?
A. Maternal infection is a usual cause of preterm labor
B. Less number organisms needed for infection in preterms
C. PROM predisposes to preterm labor
D. All of the above”

A

D

How well did you know this?
1
Not at all
2
3
4
5
Perfectly
32
Q

“True about BPD in neonates?

a. Requires< 10% of PO2 at 59 days
b. Male predominant
c. It is inversely related to gestational age”

A

C

“Nelson 21st p936-937
BPD is a clinical pulmonary syndrome that develops in the majority of extremely preterm infants and is defined by a prolonged need for respiratory support and supplemental oaxygen. Almost 60% of infants born at <=28 wk gestation will develop BPD, and the incidence of BPD increases inversely with gestational age.

BPD is diagnosed when a preterm infant requires supplemental oxygen for the 1st 28 postnatal days, and is further classified at 36wk PMA according to the degree of O2 supplementation.

Neonates recieving PPV or >=30 supplemental O2 at 36wk PMA or at discharge (whichever occurs first) are diagnosed as having severe BPD

Those requiring 22-29% supplemental O2 have moderate BPD, and those who previously required O2 supplemenation for at least 28 days but are currently breathing room air have mild BPD. “

How well did you know this?
1
Not at all
2
3
4
5
Perfectly
33
Q
"What causes severe jaundice in newborns?
A. Bottle feeding
B. Jaundice before 24 hours
C. Both of the above
D. None of  the above"
A

B

How well did you know this?
1
Not at all
2
3
4
5
Perfectly
34
Q

“Neonate presents with bilious vomiting and abdominal distention. On x-ray, patient had distended bowel loops with ground glass appearance of intestines and microcolon, cecum fixed in RLQ. What is the condition?

a. Meconium ileus
b. Malrotation
c. Hirschsprung disease
d. Volvulus”

A

A

“Nelson 21st p949-950 Meconium ileus
Clinically, neonates present with intestinal obstruction with or without perforation. Abdominal distension is prominent, and vomiting, often bilious, becomes persistent, although occasionally inspissated meconium stools may be passed shortly after birth.

Plain radiographs reveal small bowel obstruction. Air-fluid levels may not be apparent because of the thickened meconium. In contrast to the generally evenly distributed abdominal loops above an atresia, the loops may vary in width and are not as evenly filled with gas. At points of the heaviest meconium concentration, the infiltrated gas may create a bubbly, granular appearance. “

How well did you know this?
1
Not at all
2
3
4
5
Perfectly
35
Q

“Mother has seizure disorder. Also has bipolar disorder taking lithium. She asks you about breastfeeding. What advise can you give?

a. Discontinue lithium
b. Do not discontinue lithium
c. Add another medication for bipolar disorder
d. None of the above “

A

A

Lithium is teratogenic, associated with Ebstein anomaly of the heart

How well did you know this?
1
Not at all
2
3
4
5
Perfectly
36
Q

Associated with oligohydramnios.

A

RENAL AGENESIS

“Nelson 21st p880 Table 114.4 Conditions associated with disorders of amniotic fluid volume

Oligohydramnios

  • amniotic fluid leak with rupture of membranes
  • IUGR
  • fetal abnormalities (particularly GU abnormalities
  • twin-twin transfusion (donor)
  • fetal akinesia syndrome
  • prune-belly syndrome
  • pulmonary hypoplasia
  • amnion nodosum
  • indomethacin
  • ACE-I or ARB

Polyhydramnios

  • Congenital anomalies - CNS, TEF, intestinal atresia, spina bifida, cleft lip or palate, CCAM, diaphragmatic hernia
  • Syndromes - achondroplasia, Trisomy 18, 21, TORCH, hydrops, Bartter
  • Other - DM, twin-twin transfusion (recepient), fetal anemia, heart failure, congenital nephrotic syndrome, neuromuscular disease, chlyothorac, teratoma, idiopathic”
How well did you know this?
1
Not at all
2
3
4
5
Perfectly
37
Q

“A case of twins. Twin A has hematocrit of 70%. Twin B has hematocrit of <40%. Which statement is true?
A. Twin B is at risk for respiratory distress
B. Twin A is at risk for hyperbilirubinemia
C. Both of the above
D. None of the above”

A

C

How well did you know this?
1
Not at all
2
3
4
5
Perfectly
38
Q
"Mother gave birth. Baby admitted at NICU. Nurses claim baby has high-pitch cry. With jitteriness and irritability. This is due to what substance abuse?
A. Cocaine
B. Heroin
C. Marijuana
D. Alcohol abuse"
A

B

“Nelson 21st p976
Neonatal abstinence syndrome is the clinical diagnosis given to infants who experience withdrawal signs after in-utero exposure to opioids.

The clinical signs of NAS result from CNS hyperexcitability and autonomic instability. NAS signs can begin within 24 hr of birth after heroin exposure. Tremors, poor feeding, excesive crying, poor sleeping, and hyperirritability are the most prominent signs. Other signs include sneezing, yawning, hiccups, myoclonic jerks, skin breakdown and abrasions, vomiting, loose stools, nasa stuffiness, and seizures in the most severe cases. “

How well did you know this?
1
Not at all
2
3
4
5
Perfectly
39
Q

Neonate delivered at home, purely breastfed developed bleeding? What to do?

A

Give vitamin K

Diagnosis is VKDB

How well did you know this?
1
Not at all
2
3
4
5
Perfectly
40
Q

“Case on a newborn discharged <48 hours, which should be fulfil led prior to early discharge .
A. No jaundice
B. Passage of urine and at least one stool
C. Both of the above
D. None of the above”

A

B

”"”Prev Ped 2018 p15 Appendix 2. Discharge and followup of healthy term newborns

The Philippine Society of Newborn Medicine lists the following minimum criteria for discharging newborns within 48hrs

  1. Uncomplicated antepartum, intrapartum, and postpartum courses for both mother and newborn
  2. Vaginal delivery, singleton, completed 37 weeks AGA
  3. Normal and stable VS during the preceding 12hrs
  4. Has urinated and passed at least one stool
  5. Has documented proper latch, milk transfer, swallowing, infant satiety and absence of nipple discomfort
  6. Normal PE
  7. No evidence of significant jaundice in first 24hrs of life
  8. Educability and ability of parents to care for their child (recognize signs of illness, care of the umbilical cord/skin/genitalia, maternal confidence in feeding her infant and parents’ understanding of the importance of ffup visit or emergency consultation
  9. Must followup within the next 48hrs”””
How well did you know this?
1
Not at all
2
3
4
5
Perfectly
41
Q

“Initiation of first breath

a. Increased Pa02
b. Increased pC02
c. Increased pH
d. Decreased pCo2”

A

B

How well did you know this?
1
Not at all
2
3
4
5
Perfectly
42
Q

“True of neonatal NEC
A. Age of onset is inversely related to age of gestation
B. Bacterial infection is the greatest risk factor
C. More common in term infants
D. None of the above”

A

A

“Nelson 21st p951
Incidence and case fatality rates increase with decreasing birthweight and gestational age.

3 major risk factors have been implicated: prematurity, bacterial colonization of the gut, and formula feeding. NEC develops primarily in premature infants with exposure to metabolic substrate in the context of immature intestinal immunity, microbial dysbiosis, and mucosal ischemia

The greatest risk factor for NEC is prematurity. NEC rarely occurs before the initiation of enteral feeding and is much less common in infants fed human milk. Aggressive enteral feeding may predispose to the development of NEC.”

How well did you know this?
1
Not at all
2
3
4
5
Perfectly
43
Q

“You assisted a delivery of full term baby at NICU. At 15 minutes of life, patient was crying, pink, but blue extremities, good tone, CR more than 100. What will you do first?
A. Do APGAR scoring
B. Dry and do routine newborn care
C. Do Ballards scoring”

A

B

How well did you know this?
1
Not at all
2
3
4
5
Perfectly
44
Q

Case of HIE. Most devastating complication?

A

Permanent damage to CNS

How well did you know this?
1
Not at all
2
3
4
5
Perfectly
45
Q

“Eye screening by paediatrician should be done
A. At birth
B. Before discharge
C. On follow up”

A

A

How well did you know this?
1
Not at all
2
3
4
5
Perfectly
46
Q

“Preterm born at 30 weeks AOG. When is the best time to do ROP screening timing
A. 2 weeks after birth
B. 4 weeks after birth
C. 8 weeks after birth”

A

B

”"”Prev Ped 2018 p21
Criteria for screening retinopathy of prematurity
1. 32 weeks or younger AOG
2. 1500g or lighter
3. Older than 32 weeks or heavier than 1500g but with stormy medical course at the NICU”””

How well did you know this?
1
Not at all
2
3
4
5
Perfectly
47
Q

“Ominous sign of RDS
A. Grunting
B. Respiratory distress
C. Apnea”

A

C

“Nelson 21st p932
Characteristically, tachypnea, prominent (often audible) expiratory grunting, intercostal and subcostal retractions, nasal flaring, and cyanosis are noted. Breath sounds may be normal or diminished with a harsh tubular quality, and on deep inspiration fine crackles can be heard.

The natural course of untreated RDS is characterized by progressive worsening of cyanosis and dyspnea. If the condition is inadequately treated, blood pressure may fall, cyanosis and pallor increase, and grunting decreases or disappears as the condition worsens.

Apnea and irregular respirations are ominous signs requring immediate intervention. Respiratory failure may occur in infants with rapid progression of the disease. “

How well did you know this?
1
Not at all
2
3
4
5
Perfectly
48
Q

“Which of the following ElNC practices encourage early breastfeeding initiation?
A. Immediate & thorough drying of the baby
B. Properly timed cord clamping & cutting
C. First embrace of the mother & baby
D. Non-drug pain relief for the mother before offering anesthesia”

A

C

”"”Prev Ped 2018 p14
The recommended EINC practice for immediate care of the normal newborn are a series of time-bound interventions at the time of birth that can be enforced immediately in all health care settings. It emphasizes the step by step performance of a sequence of four core actions which are:

  1. Immediate and thorough drying of the newborn
  2. Early skin to skin contact between mother and newborn
  3. Properly timed cord clamping and cutting
  4. Nonseparation of newborn and mother for early breastfeeding.

These time sensitive interventions should not be pre-empted nor undermined by other interventions. Unnecessary interventions in newborn care include routine separation from the mother, foot printing, appplication of various substances to the cord, and giving pre-lacteals or artificial milk formula or other breast milk substitutes.”””

How well did you know this?
1
Not at all
2
3
4
5
Perfectly
49
Q

“After immediate & thorough drying of the newborn, what is the next step in EINC which also contributes to providing heat to the infant?
A. Early skin to skin contact between the mother and baby
B. Proper swaddling of the newborn with clean linen
C. Early bathing of the newborn with warm water
D. Exposure of the newborn to a radiant heat warmer”

A

A

”"”Prev Ped 2018 p14
The recommended EINC practice for immediate care of the normal newborn are a series of time-bound interventions at the time of birth that can be enforced immediately in all health care settings. It emphasizes the step by step performance of a sequence of four core actions which are:

  1. Immediate and thorough drying of the newborn
  2. Early skin to skin contact between mother and newborn
  3. Properly timed cord clamping and cutting
  4. Nonseparation of newborn and mother for early breastfeeding.

These time sensitive interventions should not be pre-empted nor undermined by other interventions. Unnecessary interventions in newborn care include routine separation from the mother, foot printing, appplication of various substances to the cord, and giving pre-lacteals or artificial milk formula or other breast milk substitutes.”””

How well did you know this?
1
Not at all
2
3
4
5
Perfectly
50
Q
"A newborn was delivered via emergency caesarian section due to fetal distress. At birth, he was noted to be pale, non- reactive to stimuli, with slightly flexed extremities, a HR of 70 beats/ minute, & RR of 40 breaths/ minute. What is his APGAR score?
A. 3
B. 4
C. 5
D. 6"
A

B

"""A - 0  
P - 1 
G - 0 
A - 1 
R - 2 

= 4

Nelson 21st p872 Table 113.2 Apgar evaluation of newborn infants

Heart rate
0 - absent
1 - below 100
2 - over 100

Respiratory effort
0 - absent
1 - slow, irregular
2 - good, crying

Muscle tone
0 - limp
1 - some flexion of extremities
2 - active motion

Response to catheter in nostril (tested after the oropharynx is clear)
0 - no response
1 - grimace
2 - cough or sneeze

Color
0 - blue, pale
1 - body pink, extremities blue
2 - completely pink

”””

How well did you know this?
1
Not at all
2
3
4
5
Perfectly
51
Q
"A term newborn was gasping & apneic at birth, with a HR of 80 beats/ minute. What is the most appropriate intervention within the first minute of life
A. Intubate as soon as possible
B. Do vigorous stimulation
C. Give oxygen support at 5 LPM
D. Start positive pressure ventilation"
A

D

”"”Nelson 21st p926
In term infants after stimulation, if no respirations are noted, or if the heart rate is <100 BPM, PPV should be given through a tightly fitted and appropriately sized bag-mask device. PPV should be initiated at pressures of approximately 20cm H2O at a rate of 40-60 breaths per minute, initially with 21% FiO2 for full term infants.
“””

How well did you know this?
1
Not at all
2
3
4
5
Perfectly
52
Q

“What is the most important & effective action in neonatal resuscitation?
A. Perform chest compressions to improve circulation
B. Ventilate the baby’s lungs to prevent respiratory failure
C. Administer fluids for adequate volume requirement
D. Administer epinephrine to prevent cardiac failure”

A

B

”"”Nelson 21st p926
Failure to initiate or sustain respiratory effort is fairly common at birth, with 5-10% of births requiring some intervention. Infants with primary apnea respond to stimulation by establishing normal breathing. Infants with secondary apnea require some ventilatory assistance in order to establish spontaneous respiratory effort. Secondary apnea usually originates in the CNS as a result of asphyxia or peripherally because of neuromuscular disorders.

The steps in neonatl resuscitation follows the ABCs: anticipate and establish a patent airway by positioning the baby with the head slightly extended, sniffing position, and suctioning if secretions are blocking the airway; B initiae breathing first by using tactile stimulation, followed by PPV with a bag-mask device and ETT insertion should the baby remain apneic or PPV is not achieving effective ventilation; and C, maintain the circulation with chest compressions and medications, if needed. “””

How well did you know this?
1
Not at all
2
3
4
5
Perfectly
53
Q
"A 2-day old female, term, delivered via NSD, was discharged on the 36TH hour of life. She was purely breastfed with good suck & activity. However, jaundice was noted on the 30TH hour of life. To assess her transition to life at home & monitor for other problems, she should be brought back to her pediatrician after how many days?
A. 1-3
B. 4-6
C. 7-9
D. 10-12"
A

A

“Prev Ped 2018 p15 Appendix 2. Discharge and followup of healthy term newborns

The Philippine Society of Newborn Medicine lists the following minimum criteria for discharging newborns within 48hrs

  1. Uncomplicated antepartum, intrapartum, and postpartum courses for both mother and newborn
  2. Vaginal delivery, singleton, completed 37 weeks AGA
  3. Normal and stable VS during the preceding 12hrs
  4. Has urinated and passed at least one stool
  5. Has documented proper latch, milk transfer, swallowing, infant satiety and absence of nipple discomfort
  6. Normal PE
  7. No evidence of significant jaundice in first 24hrs of life
  8. Educability and ability of parents to care for their child (recognize signs of illness, care of the umbilical cord/skin/genitalia, maternal confidence in feeding her infant and parents’ understanding of the importance of ffup visit or emergency consultation
  9. Must followup within the next 48hrs”
How well did you know this?
1
Not at all
2
3
4
5
Perfectly
54
Q
"The most common causes of readmissions among newborns discharged very early from birthing facilities are hyperbilirubinemia, sepsis, dehydration, and
A. Bowel disturbances
B. Hepatitis infection
C. Ophthalmic ointment reaction
D. Missed congenital anomalies"
A

D

How well did you know this?
1
Not at all
2
3
4
5
Perfectly
55
Q
"At what age in weeks is a physiologic decrease in hemoglobin content observed in term infants?
A. 1-4
B. 4-8
C. 8-12
D. 12-16"
A

C

“Nelson 21st p2516
At birth, normal full-term infants have higher hemoglobin (Hb) levels and larger red blood cells (RBCs) than do older children and adults. However, within the 1st wk of life, a progressive decline in Hb level begins and then persists for 6-8 wk. The resulting anemia is known as the physiologic anemia of infancy.”

How well did you know this?
1
Not at all
2
3
4
5
Perfectly
56
Q

“Which of the following features is TRUE of breast milk jaundice?
A. It is usually seen within the first week of life.
B. If breastfeeding is continued, the bilirubin levels gradually decreases.
C. If breastfeeding is discontinued, the bilirubin levels falls rapidly but usually returns to high levels with resumption of breastfeeding.
D. There is significant elevation of conjugated bilirubin.”

A

B

“Breastfeeding jaundice

    - 3rd-4th DOL
    - Inadequate supply of breastmilk leading to increased enterohepatic circulation
    - Tx: Increase breastfeeding to 8-10x/day 

Breast milk jaundice

    - 1st-2nd week of life
    - Glucoronidase in breast milk increases enterohepatic circulation
    - Tx: Increase breastfeeding frequency, phototherapy"
How well did you know this?
1
Not at all
2
3
4
5
Perfectly
57
Q
"What is the most important neonatal factor predisposing to infection?
A. Low birth weight
B. Maternal infection
C, Septic delivery
D. Meconium aspiration"
A

A

”"”Nelson 21st p1005 HAI
Premature and VLBW infants often have prolonged hospitalizations are are particularly prone to healthcare-acquired infections because of their inefficient innate immunity, deficient skin barriers, presence of indwelling catheters and other devices, and prolonged endotracheal intubation.

Nelson 21st p1012 Perinatal infections
Factors influencing which colonized infants will experience disease are not well understoon, but include prematurity, underlying illness, invasive procedures, inoculum size, virulence of the infecting organism, genetic predisposition, the innate immune system, host response, and transplacental maternal antibodies. “””

How well did you know this?
1
Not at all
2
3
4
5
Perfectly
58
Q
"A term infant was born after a normal pregnancy. However, the delivery was complicated by marginal placental separation . On the 12TH hour of life, he passed out bloody meconium although he appeared to be well. Which of the following tests should be prioritized to determine the cause of bleeding?
A. Barium enema
B. A PT test
C. Gastric lavage with normal saline
D. Upper GI series"
A

B


Nelson 21st p912
Vomitus containing dark blood is usually a sign of serious illness, but the benign possibility of swallowed maternal blood associated with the delivery process should also be considered. Tests for maternal vs fetal hemoglobin (pt tests) can help discriminate between these possibilities. “

How well did you know this?
1
Not at all
2
3
4
5
Perfectly
59
Q

“What is the recommended storage period for breast milk?
A. 4 hours refrigerated at 4 C
B. 24 hours at room temperature of< 25 C
C. 8 days at the freezer compartment of a 2-door refrigerator
D. 6 months at deep freezer with constant temperature of -20 C”

A

D

”"”Prev Ped 2018 p26

Breastmilk storage period

  1. Room temperature (<25C) - 4 hours
  2. Room temperature (>25C) - 1 hour
  3. Referigerator (4C) - 8 days
  4. Freezer compartment of a 1 door refrigerator - 2 weeks
  5. Freezer compartment of a 2 door refrigerator - 3 months
  6. Deep freezer with a constant temperature -20C - 6 months”””
How well did you know this?
1
Not at all
2
3
4
5
Perfectly
60
Q

“Vitamin A has a role in mucosal re-epithelialization in what viral infection?

a. Roseola
b. Rotavirus
c. Varicella
d. HSV”

A

C

How well did you know this?
1
Not at all
2
3
4
5
Perfectly
61
Q
"Vitamin A is?
A. Water soluble
B. Fat soluble
C. No role with growth and development
D. Cant remember"
A

B

Fat soluble vitamins = ADEK

62
Q
"Vitamin-responsive seizure. What is the vitamin? 
A.B6
B.B12
C. B1
D. B2"
A

A

“Nelson 21st p370 B6 pyridoxine
The vitamin B6 deficiency symptoms seen in infants are listlessness, irritability, seizures, vomiting, and failure to thrive. Peripheral neuritis is a feature of deficiency in adults but is usually not seen in childrens. Skin lesions include cheilitis, glossitis, and seborrheic dermatitis around the eyes, nose, and mouth. Microcytic anemia can occur in infants but is not common. Oxaluria, oxalic acid bladder stones, hyperglycinemia, lymphopenia, decreased antibody formation, and infections are also associated with vitamin B6 deficiency.

Nelson 21st p372 B12 Cobalamine
The hematologic manifestations of vitamin B12 deficiency are similar to manifestations of folate deficiency. Irritabilty, hypotonia, developmental delay, developmental regression, and involuntary movements (coarse tremors) are the most common neurologic symptoms in infants.

Nelson 21st p365-366 B1 thiamine
Early symptoms of thiamine deficiency are nonspecific, such as fatigue, apathy, irritability, depression, drowsiness, poor mental concentration, anorexia, nausea, and abdominal discomfort. As the condition progresses, more specific manifestations of beriberi develop, such as peripheral neuritis, decreased deep tendon reflexes, loss of vibration sense, tenderness and cramping of the leg mucles, heart failure, and psychologic disturbances.

Patients can have ptosis of the eyelids and atrophy of the optic nerve. Hoarseness or aphonia caused by paralysis of the laryngeal nerve is a characteristic sign.

The classic triad of Wernicke encephalopathy - mental status changes, ocular signs, and ataxia - is rarely reported in infants or young children with severe deficiency.

Death from thiamine deficiency is usually seconary to cardiac involvement

Nelson 21st p366 B2 Riboflavin
Clinical features of nutritional riboflavin deficiency include cheilosis, glossitis, keratitis, conjunctivitis, photophobia, lacrimation, corneal vascularization, and seborrheic dermatitis “

63
Q

Malnutrition management

A

Rehabilitation and stabilization

“Nelson 21st p338
The aim of the stabilization phase is to repair cellular function, correct fluid and electrolyte imbalance, restore homeostasis, and prevent death from the interlinked triad of hypoglycemia, hypothermia, and infection. The aim of the rehabilitation phase is to restore wasted tissues (ie. catch up growth) “

64
Q

“Chronic malnutrition
A. Ht for Wt
B. Wt for It
C. BMI”

A

B

“Prev Ped 2016 p33 Figure 3. Z score interpretation

Length/Height for age

  • Below -2: Stunted
  • Below -3: Severely stunted

Weight for age

  • Below -2: Underweight
  • Below -3: Severely underweight

Weight for length/height

  • Above 3: Obese
  • Above 2: Overweight
  • Above 1: Possible risk for overweight
  • Below -2: Wasted
  • Below -3: Severely wasted

BMI for age - Above 3: Obese

  • Above 2: Overweight
  • Above 1: Possible risk for overweight
  • Below -2: Wasted
  • Below -3: Severely wasted”
65
Q

What should not be management of acute malnutrition

A

Diuretics and high protein diet -

May cause refeeding syndrome

66
Q

Components of metabolic syndrome

A

Insulin resistance, compensatory hyperinsulinemia, obesity, dyslipidemia, hypertension

”"”The metabolic syndrome consists of central obesity,
hypertension, glucose intolerance, and hyperlipidemia

(Nelsons 21st p351)”””

67
Q

“How to prevent obesity?
A. Restrict all media
B. Lessen meals
C. Eat with the family at fixed and regular times”

A

C

“PPS Policy Statements 2009 vol 1 no 1 p3-4. Obesity in children and adolescents

It is the position of the PPS to adopt the following (additional) preventive measures.

  1. Breastfeeding for at least 6 months and onwards, proper complementary feeding
  2. Nutrition
    - Home cooked meals should be encouraged as opposed to eating out in restaurants
    - Avoidance of fast food
  3. Physical activity
    - Engage in regular exercise
    - Minimize viewing of television
    - Encourage family support

Nelson 21st p357 Table 60.8 Proposed suggestions for preventing obesity

Family

  • Eat meals aas a family in a fixed place and time
  • Do not skip meals, especially breakfast
  • Do not allow television during meals
  • Use small plates, keep serving dishes away from the table
  • Avoid unnecessary sweet or fatty foods and sugar-sweetened drinks
  • Remove televisions from children’s bedrooms, restrict times for TV viewing and video games
  • Do not use food as a reward”
68
Q

Case of periorbital swelling, edema, limitation of motion and pain. Diagnosis?

A

Orbital cellulitis

“Preseptal cellulitis - no proptosis, normal ocular movements, normal pupil function
Orbital cellulitis - proptosis, limitation of eye movement, edema of the conjunctiva, inflammation and swelling of the eyelids with potentially decreased visual acuity “

69
Q
"Medicine that results to rhinitis medicamentosa?
A. Steroids
B. Antihistamines
C. Topical adrenergics
D. Phenelyphrine"
A

C

”"”Rhinitis medicamentosa (RM), also known as ‘rebound congestion’ is inflammation of the nasal mucosa caused by the overuse of topical nasal decongestants. It may be also caused by oral beta-adrenoceptor antagonists, antipsychotics, oral contraceptives, and antihypertensives. It classifies as a subset of drug-induced rhinitis.

Topical nasal decongestants can classify as either beta-phenylethylamine derivatives (ephedrine, phenylephrine) or imidazoline (naphazoline, oxymetazoline, xylometazoline) derivatives

(Rhinitis medicamentosa NCBI 2021)”””

70
Q
"Patient with purulent nasal discharge with gray grapelike masses on squeezed through the turbinates and septum
A. Foreign body
B. Dermoid
C. Nasal polyp
D. Fibrous dysplasia"
A

C

“Nelson 21st p2184. Nasal polyps
Obstruction of the nasal passages is prominent, with associated hyponasal speech and mouth breathing. Profuse unilateral mucoid or mucopurulent rhinorrhea may also be present. An examination of the nasal passages shows glistening, gray, grape-like masses squeezed between the nasal turbinates and the septum. “

71
Q
"Case of ear pulling in a toddler and on pe, nonbulging tympanic membrane but with edema and purulent earwax.
A. Otitis externa
B. Otitis media
C. Otitis interna
D. None of the above"
A

B

”"”Otitis externa - Acute ear pain, edema of ear canal, erythema, thick, clumpy otorrhea; cerumen is white and soft in consistency

Acute otitis media

  1. Moderate to severe bulging of the TM or new onset otorrhea not caused by otitis externa
  2. Mild bulging of the TM and recent (<48hr) onset of ear pain or intense TM erythema

Otitis media with effusion - bulging of the TM is absent or slight, erythema absent or slight

Signs of middle ear effusion

  1. White, yellow, amber, blue TM discoloration
  2. Opacification other than that caused by scarring
  3. Decreased or absent mobility

(Nelson 21st p3421)”””

72
Q

“Persistent bilateral otitis media with effusion for 3 months

a) Antibiotics with corticosteroids
b) Antibiotics with decongestants, anti-histamines, corticosteroids
c) Antibiotics with decongestants
d) Myringotomy with ventilation tube”

A

D

“Nelson 21st p3428
In some studies, antimicrobials have demonstrated some efficacy in resolving OME, presumably because they help eradicate nasopharyngeal infection, unapparent middle-ear infection, or both. The most significant effects of antibiotics for OME have been shown with treatment durations of 4wk and 3mo.

The efficiacy of corticosteroids in the treatment of OME has shown to be short-term. Therefore the risk-to-benefit ratio for steroids is such that they are no longer recommended for treatment of OME. Antihistamine-decongestant combinations are not effective for treating children with OME and are not indicated in management of OME. Antihistamines alone, decongestants alone, and mucolytic agents are also ineffective and are not recommended for treating patients with OME.

When OME persists despite an ample period of watchful waiting, generally 3-6 mo or perhaps longer in children with unilateral effusion, consideration of surgical intervention with myringotomy and insertion of typmanostomy tubes is appropriate. “

73
Q

“Treatment of choice for otitis media

a) Macrolides
b) Amoxicillin
c) TMP-SMX
d) Co-amoxiclav”

A

B

“Nelson 21st p3425
Amoxicillin remains the drug of first choice for uncomplicated AOM under many circumstances because of its excellent record of safety, relative efficacy, palatability, and low cost.

For children with allergic reactions in which cross-reactivity with cephalosporins is less of a concern, first line therapy with cefdinir would be an appropriate choice.

Resistance to TMP-SMX my many strains of nontypeable HiB and S. pneumonia and a reported high clinical failure rate in children with AOM treated initially with this antimicrobial argue against its use. Similarly, increasing rates of macrolide resistance argue against the efficacy of azithromycin.

Although not approved by the FDA for use in children, many clinicians have used quinolones in this patient population. “

74
Q

“What is the most common cause of stridor in children?

a. Vocal cord paralysis
b. Viral infection
c. Laryngomalacia”

A

C

“Nelson 21st p2207
Laryngomalacia accounts for 45% to 75% of congenital laryngeal anomalies in children with stridor. Stridor is inspiratory, low pitched, and exacerbated by any exertion: crying, agitation, or feeding. “

75
Q
"Case of kerosene ingestion. On PE, vesicular breath sounds and flattened diaphragm, with tachypnea. What will you do.
A. Observe for 6-8hrs then mgh
B. Mgh
C. Admit and repeat cxr after 24 hrs.
D. None of the above"
A

C

”"”Toxicity of kerosene is due to its local irritating effect and systemic effects. There is a high aspiration potential due to its low viscosity. CNS depressant effects may occur, and seizures may occur due to hypoxia

Ingestion of >30ml with vomiting presents a higher risk for development of pulmonary toxicity

Major complications

  1. Aspiration pneumonia
  2. Gastritis
  3. Hypoprothrombinemia (low PT)
  4. Seizures

(Algorithms of Common Poisonings 4th ed)”””

76
Q

“Which drugs and chronic complications are inappropriately paired?

a. Carbamazepine - Steven Johnson like syndrome
b. Phenytoin - Rickets
c. VPA - hyperammonemia
d. Acetazolamide - Tics”

A

D

“Nelson 21st p3106 Table 611.10. Some adverse effects of antiepileptic drugs

Acetazolamide

  • Nuisance: dizziness, polyuria, electrolyte imbalance
  • Serious: SJS, renal calculi

Benzodiazepines

  • Nuisance: dose-related neurotoxicity (drowsiness, sedation, ataxia), hyperactivity, drooling, increased secretions
  • Serious: apnea

Carbamazepine

  • Nuisance: tics, transient leukopenia, hyponatremia, weight gain, nausiea, dizziness
  • Serious: SJS, agranulocytosis, aplasic anemia, liver toxicity

Leviteracetam

  • CNS adverse effects: somnolence, asthenia, dizziness, but usually less than other AEDs
  • In children: anger, irritability, other behavioral symptoms
  • In adults: depressive mood

Oxcarbazepine
- Somnolence, headache, dizziness, nausea, apathy, rash, hypertrichosis, gingival hypertrophy, hypernatremia

Phenobarbital and other barbiturates

  • Nuisance: neurotoxicity, insomnia, hyperactivity, signs of distractability, fluctuation of mood, agressive outbursts, rickets
  • Serious: liver toxicity, SJS

Phenytoin and other hydantoins

  • Nuisance: gingival hyperplasia, coarsening of the facies, hirsutism, cerebellovestibular symptoms (nystagmus and ataxia), rickets
  • Serious: SJS, liver toxicity

Valproic acid

  • Nuisance: weight gain, hyperammonemia tremor, alopecia, menstrual irregularities
  • hepatic and pancreatic toxicity “
77
Q

“Most common cause of medicinal poisoning the Philippines?

a. Paracetamol
b. lsoniazid
c. Ferrous sulfate
d. Aspirin”

A

A

“PPS Policy Statements 2005 vol 1 no 6 p37. Medicinal poisoning
Most of the drugs in the essential drugs list are also poisons when taken in large doses. Paracetamol is included in the list of essential drugs, however, local data from NPCIS from 1993-1996 showed that paracetamol was one of the top 10 poisons in the Philippines.”

78
Q

“Paracetamol toxicity. Mother gave her child 10 ml q 2hours. Noted nausea, abdominal pain, vomiting, diaphoresis

a. 75 mg/kg
b. 100 mg/kg
c. 125 mg/kg
d. 150 mg/kg”

A

D

“PPS Policy Statements 2005 vol 1 no 6 p38. Medicinal poisoning
Significant individual susceptibility to the hepatototoxic effects of paracetamol includes age, diet, nutritional, and metabolic states of the patient. Toxic dose is at 150mg/kg

Acute paracetamol toxicity can be described in different phases:

  1. Phase 1 (30 minutes to 24 hours) - nausea, malaise, pallor, vomiting, diaphoresis; patients may appear normal
  2. Phase 2 (24 to 72 hours) - symptomatology of Phase 1 less pronounced; right upper equadrant pain may be present secondary to hepatic damage; blood chemistries and renal functions become abnormal
  3. Phase 3 (72 to 96 hours) - hepatic necrosis, encephalopathy; coagulation, defects, jaundice, renal failure, myocardial pathology; centrilobar necrosis on liver biopsy; nausea, vomiting, death related to hepatic failure preceded by anuria and coma
  4. Phase 4 (4 days to 2 weeks) - complete resolution of hepatic dysfunction will occur if damage during phase 3 is reversible “
79
Q

“A child played on December 31, 2017. Noted garlic odor of breath plus vomiting

a. Watusi poisoning
b. Lead
c. Insecticide
d. Kerosene”

A

A

“PPS Policy Statements 2005 vol 1 no 7 p43. Watusi poisoning
The signs and symptoms of watusi poisoning include burns, burning pain in the throat and garloc odor from breath, nausea, vomiting, diarrhea, abdominal pain, and shock. “

80
Q

“As a paediatrician, what is your role in adolescent care?

a. In sexually active female, make sure to have a routine gynecologic examination
b. Assure that sexuality is normal and not to be regarded as something dirty
c. Obtain a complete and accurate sexual history to all sexually active adolescents
d. All of the above “

A

D

“PPS Policy statements 2005 vol 1 no 1 p4. Reproductive health education among adolescents

Role of Physicians

  1. Physicians should equip themselves with adequate knowledge and information regarding reproductive heatlh issues and sexuality, especially on gender sensitive issues, physical and sexual abuse
  2. Physicians should obtain a complete and accurate sexual history from all adolescent patients
  3. Physicians should ensure that all sexually active female adolescents undergo a routine gynecologic examination
  4. Physicians should encourage, and if necessary, guide parents to discuss reproductive health and sexuality with their children
  5. Physicians should provide confidential, ethically sensitive, and nonjudgmental counsling to all adolescents regarding different issues in sexuality
  6. Physicians should be able to provide appropriate referrals to adolescents with special concerns and issues
  7. Physicians should be aware of adolescent reproductive health education programs in schools, and if necessary, serve as consultants for questions on reproductive anatomy and physiology
  8. Physicians should make handouts, brochures, leaflets, and flyers on adolescent reproductive heatlh and sexuality available in their offices or clinics
  9. Physicians should screen all sexually active adolescents for child abuse
  10. Physicians should screen adolescents for other risk factors like smoking, alcohol drinking, drug abuse, violence and depression. These factors correlate with early sex initiation among adolescents “
81
Q

Backpack guideline

A

The weight of the backpack is no more than 10%of the child’s body weight with both straps worn on and never more than four inches below the waistline

“PPS Policy Statements 2006 vol 1 no 3 p14. Backpacks and children
Parents should make sure that:
1. The weight of the backpack used by their child is no more than 10% of the child’s body weight
2. That the backpacks are properly worn, with both straps on and never more than 4 inches below the waistline

Parents should select backpacks that are appropriate in size, made of lighter material, with compartments and wide-soft S shaped padded straps that fit the back of the child. If possible, backpacks chosen should also have waist or chest straps with built in back support and lumbar pillow. “

82
Q

“Source of noise that is most damaging

a. Motorcycle
b. Recreational
c. Occupaitonal
d. Construction”

A

A

“PPS Policy statements 2006 vol 1 no 5 p21-22. Noise in the environment
Street traffic is the most prevalent and perhaps most damaging form of noise…road noise levels measured in areas with tricycle prominence usually exceed the maximum standard of 80dBA of noise. Whereas cares produce noise levels in the range of 67-75 dB, motorcycle noise ranges from 72-83 dB but can reach levels as high as 120 dB.

Construction noise is another major source of noise pollution. Like industrial equipment, construction equipment tend to produce noise at the lower end of the frequency spectrum.

Consumer products represent a wide range of noise-producing items and as such are becoming major causes of hearing impairment among children. These products are divided among four categories: recreational (guns, video arcades, fireworks), hobbies/workshop (chain saws, lawn mowers), household (garbage disposal, food blenders, vacuum cleaners), and music (concerts, personal music). Among these, guns are the loudest with peak intensities well above 120 dB, followed by hobby motorcycles and rock concerts, both of which can approach 120 dB at their peak intensity levels.

PPS Policy Statements 2006 vol 1 no 6 p27. Occupational noise exposure
The noise present in the workplace is defined as occupational noise. The workplace can be the office for an employee, a construction site for carpenters, a mining area for miners, and so forth. “

83
Q

“How to prevent obesity

a. Introduce complementary food at 4 months
b. Advocate exclusive breastfeeding for at least 6 months to 3 years
c. Both of the above
d. None of the above”

A

B

“PPS Policy Statements 2009 vol 1 no 1 p3-4. Obesity in children and adolescents

It is the position of the PPS to adopt the following (additional) preventive measures.

  1. Breastfeeding for at least 6 months and onwards, proper complementary feeding
  2. Nutrition
    - Home cooked meals should be encouraged as opposed to eating out in restaurants
    - Avoidance of fast food
  3. Physical activity
    - Engage in regular exercise
    - Minimize viewing of television
    - Encourage family support”
84
Q

“Acceptable treatment for croup

a. Cold mist
b. Antibiotics
c. Oral corticosteroids”

A

C

“Nelson 21st p2204-2205
The mainstay of treatment for children with croup is airway management and treatment of hypoxia. Treatment of the respiratory distress should take priority over any testing.

Dspite the observation that cold night air is beneficial, a Cochrane review has found no evidence supporting the use of cool mist in the emergency department for treatment of croup

Nebulized racemic epinephrine is the established treatment for moderate or severe croup. The mechanism of action is believed to be constriction of the precapillary arterioles through the B-adrenergic receptors, causing fluid resorption from the interstitial space and a decrease in the laryngeal mucosal edema. A dose of 0.25-0.5ml of 2.25% racemic epinephrine in 3ml of normal saline can be used as often as every 20 min.

The effectiveness of corticosteroids in viral croup is well established. Corticosteroids decrease the edema in the laryngeal mucosa through their anti inflammatory action. Most studies have demonstrated the efficacy of oral dexamethasone used as a single dose of 0.6mg/kg

Antibiotics are not indicated in croup. Nonprescription cough or cold medications should not be used in children younger than 6 yr of age. “

85
Q

Hospital management for croup

A

Humidified 02, racemic epinephrine, oral corticosteroid

“Nelson 21st p2204-2205
The mainstay of treatment for children with croup is airway management and treatment of hypoxia. Treatment of the respiratory distress should take priority over any testing.

Dspite the observation that cold night air is beneficial, a Cochrane review has found no evidence supporting the use of cool mist in the emergency department for treatment of croup

Nebulized racemic epinephrine is the established treatment for moderate or severe croup. The mechanism of action is believed to be constriction of the precapillary arterioles through the B-adrenergic receptors, causing fluid resorption from the interstitial space and a decrease in the laryngeal mucosal edema. A dose of 0.25-0.5ml of 2.25% racemic epinephrine in 3ml of normal saline can be used as often as every 20 min.

The effectiveness of corticosteroids in viral croup is well established. Corticosteroids decrease the edema in the laryngeal mucosa through their anti inflammatory action. Most studies have demonstrated the efficacy of oral dexamethasone used as a single dose of 0.6mg/kg

Antibiotics are not indicated in croup. Nonprescription cough or cold medications should not be used in children younger than 6 yr of age. “

86
Q
"Clubbing of digits with purulent sputum
a Bronchitis
b. Bronchiectasis
c. Pneumonia
d. Tracheitis"
A

B

“Nelson p2279
The most common complaints in patients with bronchiectasis are cough and production of copious purulent sputum. Hemoptysis is seen with some frequency. Fever can occur with infectious exacerbations. Anorexia and poor weight gain may occur as time passes.

Physical examination typically reveals crackles localized to the affected area but wheezing and digital clubbing may also occur. In severe casis, dyspnea and hypoxemia may occur. Pulmonary function studies may demonstrate an obstructive, restrictive, or mixed pattern. Typically, impaired diffusion capacity is a late finding. “

87
Q

“Bronchiolitis is caused by:

a. RSV
b. Parainfluenva
c. Rhinovirus
d. Influenza”

A

A

“Nelson 21st p2217
RSV is responsible for more than 50% of cases of bronchiolitis in most reports. Other agents include human metapneumovirus, rhinovirus, parainfluenza, influenza, bocavirus, and adenovirus. “

88
Q

“Pharyngitis, 7 episodes on amoxicillin. On PE, large dull tonsils, history of bad breath and snoring. What will you do?

a. Culture
b. Give cephalosporin
c. Obtain heterophil antibodies
d. Tonsillectomy”

A

D

“Diagnosis is chronic tonsillitis

Nelson 21st p2199-2200
Children with chronic or cryptic tonsillitis often present with halitosis, chronic sore throats, foreign-body sensation, or a history of expelling foul-tasting and foul-smelling cheesy lumps. Examination reveals tonsils of a range of sizes, often containing copious debris within the crypts. The offending organism is not usually GABHS.

Tonsillectomy alone is most commonly performed for recurrent or chronic tonsillopharyngitis. Tonsillectomy has been shown to be effective in reducing the number of infections and the symptoms of chronic tonsillitis such as halitosis, persistent or recurrent sore throats, and recurrent cervical adenitis in severely affected patients. “

89
Q

“PE finding in atelectasis
a. Increased chest excursion on affected side
b. Decreased breath sounds on affected side
C. Bulging of affected side”

A

B

“Nelson 21st p2315
Physical findings of atelectasis include limitation of chest excursion, decreased breath sound intensity, and coarse crackles. Breath sounds are decreased or absent over extensive atelectatic areas.

Massive atelectasis…The chest appears flat on the affected side, with decreased respiratory excursion, dullness to percussion, and feeble or absent breath sounds are also noted. “

90
Q

A child is playing by himself with sudden onset DOB and unable to speak. Diagnosis?

A

Foreign body ingestion

“Nelson 21st p2211
The most serious complication of foreign body aspiration is complete obstruction of the airway, which may be recognized in the conscious child as sudden respiratory distress followed by an inability to speak or cough.

History is the most important factor in determining the need for bronchoscopy. Choking or coughing episodes accompanied by new onset wheezing and asymmetric breath sounds are highly suggestive of foreign body in the airway. “

91
Q

Case of patient with inspiratory stridor. Diagnosis?

A

RSV bronchiolitis

92
Q

Case of uncomplicated pneumonia. Drug of choice?

A

Amoxicillin

“Nelson 21st p2271
For mildly ill children who do not require hospitalization, amoxicillin is recommended. With the emergence of penicillin-resistant pneumococci, high doses of amoxicillin (90mkd BID) should be prescribed unless local data indicate a low prevalence of resistance. Therapeutic alternatives include cefuroxime and amoxicillin/clavulanate. “

93
Q

“4 month old baby with PCAP. In mild respiratory distress, not 02 requiring, mild dehydration. Reason for admission?

a. 4 month old
b. Mild dehydration
c. Mild respiratory distress”

A

A

“Nelson 21st p2271. Table 428.6 Factors suggesting need for hospitalization of children with pneumonia

  • Age <6 mo
  • Immunocompromised state
  • Toxic appearance
  • Moderate to severe respiratory distress
  • Hypoxemia (oxygen saturation <90% breathing room air, sea level)
  • Complicated pneumonia (Pleural effusion, empyema, abscess, bronchopleural fistula, necrotizing pneumonia, acute respiratory distress syndrome, extrapulmonary infection (meningitis, arthritis, pericarditis, osteomyelitis, endocarditis), hemolytic uremic syndrome, or sepsis.
  • Sickle cell anemia with acute chest syndrome
  • Vomiting or inability to tolerate oral fluids or medications
  • Severe dehydration
  • No response to appropriate oral antibiotic therapy
  • Social factors (e.g., inability of caregivers to administer medications at home or follow-up appropriately”
94
Q

“5 year old with cough, case of pneumonia. Most common etiologic agent?

a. S. pneumonia
b. Chlamydophila pneumonia
c. S. aureus
d. Mycoplasma pneumonia”

A

A

“PAPP Clinical practice guidelines in the evaluation and management of community acquired pnuemonia 2021 p31

The causative agents of community acquired pnuemonia vary according to the age of the child and the setting in which the infection is acquired. Generally, viruses, notably RSV, are the most common cause of pneumonia in children younger than 5 years. S. pneumonia is the most common bacteria among all age groups. Other important bacterial causes in children younger than 5 years include H. influenza S. pyogenes, M. catarrhalis. In children 5 years and older, other important causes include Mycoplasma and Chlamydophila

Also see Nelsons 21st p2268 Table 428.3. Pneumonia etiologies grouped by age of the patient (in order of frequency) - M. pneumoniae most common in chidlren 5 years or older”

95
Q

“Patient is post cardiac surgery, hours later developed dyspnea. On chest xray, opacity on the right.

a. Chylothorax
b. Hemothorax
c. Pneumothorax
d. None of the above”

A

A

“Nelson 21st p2323
Chylothorax is a pleural collection of fluid formed by the escape of chyle from the thoracic duct or lymphatics into the thoracic cavity.

Chylothorax in children occurs most frequently because of thoracic duct injury as a complication of cardiothoracic surgery. Other cases are associated with chest injury, extracorporeal membrane oxygenation, or with primary or metastatic intrathoracic malignancy, particularly lymphoma. In newborns, rapidly increased venous pressure during delivery may lead to thoracic duct rupture. “

96
Q
"Xray with chest mass with decreased breath sounds, gastric bubble
a. Pulmonary sequestration
b. Eventration of diaphragm
C. CCAM
d. CDH"
A

D

“Nelson 21st p945
Children with CDH may also have a scaphoid abdomen and increased chest wall diameter. Bowel sounds may also be heard in the chest with decreased breath sounds bilaterally. The point of maximal cardiac impulse may be displaced away from the side of the hernia if mediastinal shift has occured. A chest radiograph and passage of a nasal gastric tube are usually sufficient to confirm this diagnosis.”

97
Q

“Smoking teenager caught by mom. What will you advise?

a. Hold allowance
b. Early curfew
c. One on one talk regarding risk and refer to smoking cessation program”

A

C

98
Q
"Patient with acute respiratory distress, what is most worrisome?
A. Tachypnea
B. Retractions
C. Bradypnea
D. Increased Bp"
A

C

Bradypnea is a sign of impending respiratory failure

99
Q
"Major factor predictive of asthma
A. Parental history of asthma
B. Wheezing not associated with colds
C. Smoking.
D. Allergic rhinitis,"
A

A

”"”Early childhood risk factors for persistent asthma

MAJOR

  1. Parental asthma
  2. Atopic dermatitis
  3. Inhalant allergen sensitization
OTHER
Allergic rhinitis
Food allergy
Food allergen sensitization
Pneumonia 
Bronchiolitis requiring hospitalization
Wheezing apart from colds
Male gender
Low birthweight
Environmental tobacco smoke exposure
Reduced lung function at birth
Formula feeding 

(Nelson 21st p1187)”””

100
Q
"Pneumonia case treated with Pen G. Improved for a few days but suddenly developed dyspnea
A. Consolidation
B. Atelectasis
C. Pleural eff
D. Pneumothorax"
A

C

”"”Nelsons 21st p2271. Table 428.6
Complications of pneumonia - pleural effusion, empyema, abscess, bronchopleural fistula, necrotizing pneumonia, ARDS, extrapulmonary infection (meningitis, arthritis, pericarditis, osteomyelitis, endocarditis), hemolytic uremic syndrome, sepsis

Nelsons 21st p2273
Complications of pneumonia are usually the result of direct spread of bacterial infection within the thoracic cavity (pleural effusion, empyema, pericarditis) or bacteremia and hematologic spread. Meningitis, endocarditis, suppurative arthritis, and osteomyelitis are rare complications of hematologic spread of pneumococcal or H. influenza type B infection. S. aureus, S. pneumoniae, and S. pyogenes are the most common causes of parapneumonic effusions and empyema. “””

101
Q
"Pneumonia with chest lag. On the lagging lung, noted with increased vocal and tactile stimulus dullness on percussion.
A..Consolidation
B. Atelectasis
C. Pleural eff
D. Pneumathorax"
A

C

”"”Consolidation - crackles, ronchi, dull percussion, increased fremitus

Atelectasis - decreased BS, dull percusion, decreased fremitus

Pleural effusion - decreased BS, dull percussion, decreased fremitus

Pneumothorax - decreased BS, resonant percussion, decreased fremitus

(USMLE First Aid)”””

102
Q

“Pleural effusion serous, studies: ph 7.28 Idh 300 tp 2.5 no organisms. What to do?
A. Insert ct tube
B Repeat cxr
C.Start ampicilin”

A

B

”"”Effusion is transudative. Do repeat CXR

Lights criteria

  1. Transudate
Serous
WBC <10,000
pH >7.2 (alkaline) 
Protein < 3.0 (low)
PF:serum protein ratio <0.5
LDH <200 (low) 
PF: serum LDH ratio <0.6 
Glucose >60 (high) 
  1. Exudate
Cloudy
WBC >50,000
pH <7.2 (acidic) 
Protein > 3.0 (high) 
PF:serum protein ratio > 0.5
LDH > 200 (high) 
PF: serum LDH ratio >0.6 
Glucose < 60 (low) 

TRANSUDATE: Alkaline, low protein, low LDH, high glucose

EXUDATE: Acidic, high protein, high LDH, low glucose

(Nelson 21st p2274)”””

103
Q
"What value is indicative of empyema in pleural fluid examination?
A. LDH <200
B. Ratio<0.6
C, Glucose < 40 mg/dl
D. None of the above"
A

D

“Nelson 21st p2276
The maximal amount of fluid obtainable should be withdrawn by thoracentesis and studied as described in Chapter 429.2. The effusion is an empyema if bacteria are present on Gram staining, the pH is <7.20, and there are >100,000 neutrophils/μL

Lights criteria

  1. Transudate
Serous
WBC <10,000
pH >7.2 (alkaline) 
Protein < 3.0 (low)
PF:serum protein ratio <0.5
LDH <200 (low) 
PF: serum LDH ratio <0.6 
Glucose >60 (high) 
  1. Exudate
Cloudy
WBC >50,000
pH <7.2 (acidic) 
Protein > 3.0 (high) 
PF:serum protein ratio > 0.5
LDH > 200 (high) 
PF: serum LDH ratio >0.6 
Glucose < 60 (low) 

TRANSUDATE: Alkaline, low protein, low LDH, high glucose

EXUDATE: Acidic, high protein, high LDH, low glucose

(Nelson 21st p2274)”

104
Q
"Cause of pulmonary edema except?
A. Increased capillary pressure
B. Increased pulmonary capillary permeability
C. Decreased oncotic pressure
D. None of the above"
A

D

“Nelson 21st p2234 Table 424.1. Etiology of pulmonary edema

  1. Increased pulmonary capillary pressure
  2. Increased capillary permeability
  3. Lymphatic insufficiency
  4. Decreased oncotic pressure
  5. Increased negative interstitial pressure
  6. Mixed or unknown causes “
105
Q

“True of pathophysiology of Asthma except
A. Airway hyperresponsiveness
B. Airway and basement membrane inflammation and edema
C. Mucus hypersecretion
D. Parenchymal inflammation”

A

D

“Nelson 21st p1188
Airway obstruction in asthma is the result of numerous pathologic processes. In the small airways, airflow is regulated by smooth muscle encircling the airway lumenl; bronchoconstriction of these bronchiolar muscular bands restricts or blocks airflow.

A cellular inflammatory infiltrate and exudates distinguished by eosinophils, but also including other inflammatory cell types (neutrophils, monocytes, lymphocytes, mast cells, basophils), can fill and obstruct the airways and induce epithelial damage and desquamation into the airway lumen. Helper T lymphocytes and other immune cells that produce proallergic, proinflammatory cytokines (IL4, IL5, IL13) and chemokines (eotaxins), mediate this inflammatoory process.

Pathogenic immune responses and inflammation may also result from a breach in normal immune regulatory process (e.g. regulatory T lymphocytes that produce IL-10 and TGF-B) that dampen effector immunity and inflammation when they are no longer needed.

Hypersensitivity or susceptibility to a variety of provocative triggers can lead to airway inflammation, AHR, edema, basement membrane thickening, subepithelial collagen deposition, smooth muscle and mucous gland hypertrophy, and mucus hypersecretion - all processes that contribute to airway obstruction.”

106
Q

“A patient with sudden onset difficulty of breathing. PE: decreased breath sounds, hyperresonant. Diagnosis?

a. Pleural effusion
b. Pneumothorax
c. Atelectasis
d. Consolidation”

A

B

“Nelson 21st p2318
The onset of pneumothorax is usually abrupt, and the severity of symptoms depends on the extent of the lung collapse and the amount of preexisting lung disease. Pneumothorax may cause dyspnea, chest pain, and cyanosis.

Usually, there is respiratory distress, with retractions, markedly decreased breath sounds, and tympanitic percussion note over the involved hemithorax. The larynx, trachea and heart may be shifted toward the unaffected side. When fluid is present, there is usually a sharply limited area of tympany above a level of flatness to percussion. The presence of bronchial breath sounds or, when fluid is present in the pleural cavity, of gurgling sounds synchronous with respirations suggests an open fistula connecting with air-containing tissues. “

107
Q

“A patient with inspiratory stridor, respiratory distress, hoarseness, and brassy cough. Diagnosis?

a. Bronchiolitis
b. Pneumonia
c. Asthma
d. Croup “

A

D

“Nelson 21st p2203
Most patients have an upper respiratory tract infection with some combination of rhinorrhea, pharyngitis, mild cough, and low grade fever for 1-3 days before the signs and symptoms of upper airway obstruction become apparent.

The child then develops the characteristic barking cough, hoarseness, and inspiratory stridor. The low grade fever can persist, although temperatures may occasionally reach 39-40C; some children are afebrile. Symptoms are characteristically worse at night and often recur with decreasing intensity for several days and resolve completely within a week. “

108
Q

“A patient presented with fever, drooling, refused to move neck. PE showed bulging on posterior pharyngeal wall. Diagnosis?

a. Tonsillitis
b. Retropharyngeal abscess
c. Epiglottitis
d. Bacterial tracheitis “

A

B

“Nelson 21st p2197
Clinical manifestations of retropharyngeal abscess are nonspecific and can include fever, irritability, decreased oral intake, and drooling. Neck stiffness, torticollis, and refusal to move the neck may also be present. The verbal child might complain of sore throat and neck pain. Other signs can include muffled voice, stridor, respiratory distress, or even obstructive sleep apnea. Physical examination can reveal bulging of the posterior pharyngeal wall, although this is present in <50% of infants with retropharyngeal abscess. Cervical lymphadenopathy may also be present.

Nelson 21st p2203
Epiglottitis is characterized by an acute rapidly progressive and potentially fulminating course of high fever, sore throat, dyspnea, and rapidly progressing respiratory obstruction…Within a matter of hours, the patient appears toxic, swallowing is difficult, and breathing is labored. Drooling is usually present, and the neck is hyperextended in an attempt to maintain the airway. The child may assume the tripod position, sitting upright and leaning forward with the chin up and mouth open while bracing the arms. The diagnosis requires visualization under controlled circumstances of a large, cherry red, swollen epiglottis by laryngoscopy. “

109
Q

“A patient presented with respiratory distress with a strong family history of asthma and allergy. Came in with high HR,
crackles and wheezes, heart gallop, murmur. What is your initial impression?
a. Asthma exacerbation
b. Pneumonia
c. Pulmonary edema
d. Bronchiolitis “

A

C

“Nelson 21st p2234
The earliest clinical signs of pulmonary edema include increased work of breathing, tachypnea, and dyspnea. As fluida accumulates in the alveolar space, auscultation reveals fine crackles and wheezing, especially in dependent lung fields. In cardiogenic pulmonary edema, a gallop may be present, as well as peripheral edema and jugular venous distension. “

110
Q
"A case of a child with 10 days colds associated with sore throat and change in color and consistency of nasal secretion
A. Rhinosinusitis
B. Sinusitis
C. Bronchitis
D. Pneumonia"
A

A

“Nelson 21st p2189
Differentiating bacterial sinusitis from a cold may be difficult, but certain patterns suggestive of sinusitis have been identified. These include:
1. Persistence of nasal congestion, rhinorrhea (of any quality), and daytime cough >=10 days without improvement
2. Severe symptoms of temperature >=39C with purulent nasal discharge for 3 days or longer
3. Worsening of symptoms either by recurrence of symptoms after an initial event or new symptoms of fever, nasal discharge, and daytime cough (double sickening)

Nelson 21st p2188
Sinusitis is another complication of the common cold. The diagnosis of bacterial sinusitis should be considered if rhinorrhea or daytime cough persists without improvement for at least 10-14 days, if acute symptoms worsen over time, or if acute signs of more severe sinus improvement such as fever, facial pain, or facial swelling develop”

111
Q
"Most common presentation of chronic asthma
a, Intermittent cough and chest pain
b. Intermittent cough and wheeze
c. Intermittent cough and dyspnea 
d. None of the above"
A

B

“Nelson 21st p1188
Intermittent dry coughing and expiratory wheezing are the most common chronic symptoms of asthma. Older children and adults report associated shortness of breath and chest congestion and tightness; younger children are more likely to report intermittent, nonfocal chest pain. Respiratory symptoms can be worse at night, associated with sleep, especially during prolonged exacerbations triggered by respiratory infections or inhalant allergies. Daytime symptoms, often linked with physical activities (exercise-induced) or play, are reported with greatest frequency in children. “

112
Q
"Which FEV1/FVC ratio will confirm asthma?
A. 0.8
B. 0.9
C. 0.54
D. 0.75"
A

A

“Nelson 21st p1191 Table 169.6. Lung function abnormalities in asthma and assessment of airway inflammation

Spirometry assesses airflow limitation

  1. Low FEV1 (relative to percentage of predicted norms)
  2. FEV1 / FVC ratio of <0.80

Bronchodilator response to inhaled B-agonist assesses reversibility of airflow limitation - main criterion consistent with asthma

  1. Increase in FEV1 >12% after inhalataion of B-agonist
  2. Increase in predicted FEV1 >10% after inhalation of B-agonist

Exercise challenge - main criterion consistent with asthma
1. Worsening of FEV1 >=15%

Daily peak expiratory flow (PEF) or FEV1 monitoring - main criterion consistent with asthma
1. Day-to-day and/or AM-to-PM variation >=20%

Exhaled nitric oxide (FeNO)

  1. A value of >20 ppb supports the clinical diagnosis of asthma in children
  2. FeNO can be used to predict response to asthma therapy:
    - <20 ppb: unlikely to respond to ICS because eosinophilic inflammation unlikely
    - 20-35 ppb: intermediate, may respond to ICS
    - >35 ppb: likely to respond to ICS because eosinophilic inflammation is likely “
113
Q
"The diagnosis of asthma in children <5 years is established by:
A. Symptom pattern
B. Chest x-ray
C. Spirometry
D. Peak flow meter"
A

A

“GINA 2021 p147
It may be challenging to make a confident diagnosis of asthma in children 5 years and younger, because episodic respiratory symptoms such as wheezing and cough are also common in children without asthma, particularly in those 0-2 years old, and it is not possible to routinely assess airflow limitation or bronchodilator responsiveness in this age grou. A probability-based approach, based on the pattern of symptoms during and between viral respiratory infections may be helpful for discussion with parents/carers. This allows individual decisions to be made about whether to give a trial of controller treatment. “

114
Q

“An 11-month old male was admitted because of acute severe asthma. Which of the following admitting orders is
INAPPROPRIATE?
A. Request for ABG.
B. salbutamol 1 nebule every 1 hour
C. Budesonide 1 nebule every 12 hours
D. Hydrocortisone 5 mg/kg/dose every 6 hours”

A

C

“Nelsons 21st p1208. Emergency department management of asthma exacerbations

Initial treatment includes supplemental oxygen, inhaled B-agonist therapy every 20 min for 1 hr, and, if necessary, oral or IV systemic corticosteroids. Inhaled ipratropium may be added to the B-agonist treatment if no significant rsponse is seen with the first inhaled B-agonist treatment. An IM injection of epinephrine or other B-agonist may be administered in severe cases. Oxygen should be administered and continued for at least 20 min after SABA administration to compensate for ventilation/perfusion abnormalities caused by SABA “

115
Q
"Which of the folowing conditions requires imaging studies to establish the diagnosis?
A Pneumonia
B Atelectasis
C Bronchitis
D.Spasmodic croup"
A

B

All other choices can be diagnosed clinically. Atelectasis is a radiographic finding.

116
Q

“3-year old male had productive cough of 3 weeks with no other sign or symptom. A week PTC, he had decreased intake
with consequent weight loss. His cough was noted to be more prominent at night. On examination, his RR was 30 BPM. He had
enlarged cervical nodes, symmetrical chest expansion, no retraction & clear breath sounds. Which of the following diseases should be considered
A. Allergy
B. Pneumonia
C. TB
D. Asthma”

A

C

“Enlarged cervical lymph nodes suggestive of tuberculous etiology.

TBIC 2016 p75
For patients below 15 years old, a presumptive PTB has any of the following

A. At least 3 of the following clinical criteria:

  1. Coughing/wheezing of 2 weeks or more, especially if unexplained
  2. Unexplained fever of 2 weeks or more after common causes such as pneumonia and malaria have been excluded
  3. Loss of weight/failure to gain weight/weight faltering/loss of appetite
  4. Failure to respond to 2 weeks of appropriate antibiotic therapy for lower respiratory tract infection
  5. Failure to regain previous state of health 2 weeks after a viral infection or exanthema
  6. Fatigue, reduced playfulness, or letheargy

B. Any 1 of the above signs and symptoms in a child who is a close contact of a known active TB case

TBIC 2016 p73
In the absence of baceteriologic evidence, a child is presumed to have active TB if 3 or more of the following criteria are present:

  1. Epidemiologic - exposure to an adult/adolescent with active TB disease
  2. Clinical - signs and symptoms suggestive of PTB
  3. Immunologic - positive TST
  4. Radiologic - abnormal chest radiograph suggestive of TB
  5. Laboratory - laboratory findings suggestive of PTB (histological, cytological, biochemical, immunological, or molecular)

GINA 2021 p148 Box 6-2 Features suggesting a diagnosis of asthma in children 5 years or younger

  1. Cough - recurrent or persistent nonproductive cough that may be worse at night or accompanied by wheezing and breathing difficulties; cough occuring with exercise, laughing, crying, or exposure to tobacco smoke, particularly in the absence of an apparent respiratory infection
  2. Wheezing - recurrent wheezing, including during sleep or with triggers such as activity, laughing, crying or exposure to tobacco smoke or air pollution
  3. Difficult or heavy breathing or shortness of breath occuring with exercise, laughing or crying
  4. Reduced activity - not running, playing, or laughing at the same intensity as other children, tires earlier during walks
  5. Past or family history - other allergic disease (atopic dermatitis, allergic rhinitis, food allergy); asthma in first degree relatives
  6. Therapeutic trial with low dose ICS and as needed SABA - clinical improvement during 2-3 months of controller treatment and worsening when treatment is stopped. “
117
Q

“The pleural fluid from an infant was purulent & the analysis showed the following results: pH 7.28, LDH 300 IU, glucose
50 g/dL, CHONS 2.5 g/dL & no microorganism on gram stain. Which of the following interventions is appropriate?
A. Insert a chest tube
B. Start anti-TB drugs
C. Shift ampicilin
D. Repeat the chest x-ray”

A

A

Purulent fluid indicates empyema. Management is CTT

“Lights criteria

  1. Transudate
Serous
WBC <10,000
pH >7.2 (alkaline) 
Protein < 3.0 (low)
PF:serum protein ratio <0.5
LDH <200 (low) 
PF: serum LDH ratio <0.6 
Glucose >60 (high) 
  1. Exudate
Cloudy
WBC >50,000
pH <7.2 (acidic) 
Protein > 3.0 (high) 
PF:serum protein ratio > 0.5
LDH > 200 (high) 
PF: serum LDH ratio >0.6 
Glucose < 60 (low) 

TRANSUDATE: Alkaline, low protein, low LDH, high glucose

EXUDATE: Acidic, high protein, high LDH, low glucose

(Nelson 21st p2274)”

118
Q

“5-year old male had productive cough of 7 days. Five days PTC, he became febrile with decreased appetite. The
persistence of symptoms prompted consultation. On examination, his RR was 35 BPM. He had symmetrical chest expansion,intercostal retractions & crackles on both lung fields. The chest x-ray showed diffuse interstitial infiltrates. CBC showed Hgb 12,Hct 0.39 PMN 30, lymph 70 & adequate platelets. Which of the following interventions should be recommended?
A.Supportive & symptomatic treatment
B. Nebulization with a short-acting bronchodilator
C. Amoxcilin at 50 mg/kg/day
D. Clarithromycin at 15 mg/kg/day”

A

A

Diagnosis is viral pnuemonia. Treatment is supportive and symptomatic treatment

119
Q
"A diagnosis of bacterial tracheitis can be established by:
A. History & PE
B. Chest xray
C. Lateral x-ray of the neck
D. Complete blood count"
A

A

“Nelson 21st p2206 Tracheitis
Typically the child has a brassy cough, apparently as part of a viral laryngotracheobronchitis. High fever and toxicity with respiratory distress can occur immediately or after a few days of apparent improvement. The patient can lie flat, does not drool, and does not hav the dysphagia assocaited with epiglottitis.

The major pathologic feature appears to be mucosal swelling at the level of the cricoid cartilage, complicated by copious thick purulent secretions, sometimes causing pseudomembranes.

The diagnosis is based on evidence of bacterial upper airway disease, which includes high fever, purulent airway secretions, and an absence of the classical findings of epiglottitis. Xrays are not needed but can show the classic findings (e.g. pseudomembranes, ragged air column). Purulent maternal is noted below the cords during endotracheal intubation. “

120
Q

“A 4-year old female was admitted because of a continuous, high-pitched, inspiratory sound. Two days PTA, she had dry cough, colds with watery nasal discharge & low-grade fever. A day PTA, the cough became more frequent accompanied by sore throat & hoarseness. Two hours PTA, she refused to feed & became irritable with the sound observed after paroxysmal coughing.
Which of the following interventions is appropriate?
A. Incision & drainage
B. Bronchoscopy
C. Clindamycin + ceftriaxone
D. Dexamethasone”

A

C

“Diagnosis is croup

Nelson 21st p2204-2205
The mainstay of treatment for children with croup is airway management and treatment of hypoxia. Treatment of the respiratory distress should take priority over any testing.

Dspite the observation that cold night air is beneficial, a Cochrane review has found no evidence supporting the use of cool mist in the emergency department for treatment of croup

Nebulized racemic epinephrine is the established treatment for moderate or severe croup. The mechanism of action is believed to be constriction of the precapillary arterioles through the B-adrenergic receptors, causing fluid resorption from the interstitial space and a decrease in the laryngeal mucosal edema. A dose of 0.25-0.5ml of 2.25% racemic epinephrine in 3ml of normal saline can be used as often as every 20 min.

The effectiveness of corticosteroids in viral croup is well established. Corticosteroids decrease the edema in the laryngeal mucosa through their anti inflammatory action. Most studies have demonstrated the efficacy of oral dexamethasone used as a single dose of 0.6mg/kg

Antibiotics are not indicated in croup. Nonprescription cough or cold medications should not be used in children younger than 6 yr of age. “

121
Q

“A toddler had acute rhinosinusitis. Which oft he following statements is TRUE?
A.The average frequency in this age group is 2-4 episodes per year.
B. The infection is usualy transmitted by ingesting the virus exhaled by an infected person.
C. The nasal symptoms appear before the cough.
D. The cough usually subsides after 6 days”

A

C

“Nelson 21st p2185-2186
Young children have an average of 6-8 colds per yr, but 10-15% of children have at least 12 infections per yr. The incidence of illness decreases with increasing age, with 2-3 illnesses per yr of by adulthood.

Viruses that cause the common cold are spread by 3 mechanisms:

  1. Direct hand contact - self-inoculation of one’s own nasa mucosa or conjunctivae after touching a contaminated person or object
  2. Inhalation of small-particle aerosols that are airborne from coughing
  3. Deposition of large particle aerosols that are expelled during a sneeze and land on nasal or conjunctival mucosa

Symptoms of the common cold vary by age and virus. In infants, fever and nasal discharge may predominate. Fever is uncommon in older children and adults. The onset of common cold symptoms typically occurs 1-3 days after viral infection. The first symptom noted is often sore or scratchy throat, followed closely by nasal obstruction and rhinorrhea. The sore throat usually resolves quickly and by the 2nd and 3rd day of illness, nasal symptoms predominate. Cough is associated with 2/3 of colds in children and usually begins after the onset of the nasal symptoms. Cough may persist for an additional 1-2 wk after resolution of other symptoms. “

122
Q

“A 5-year old female was admitted because of productive cough of 10 days. On examination, she was afebrile with RR 52
BPM. She had symmetrical chest expansion, intercostal retractions & crackles on both lung fields. Based on the risk for mortality, she has PCAP:
A. A
B. B
C. C
D. D”

A

C

“PAPP 2021 PCAP CPG p15-16
Criteria for severe pneumonia (requiring admission)

  1. Respiratory signs - cyanosis/hypoxemia, head bobbing, retractions, apena, grunting
  2. CNS signs - lethargic/stuporous/comatose/GCS <13, seizures
  3. Circulatory signs - capillary refill >3s or shock, pallor
  4. General considerations - moderate to severe malnutrition, refusal or inability to take food/drink per orem, some to severe dehydration, age <6 months
  5. Ancillary parameters - CXR or UTZ findings of consolidation, multifocal disease, moderate to large effusion, abscess, air leak; sustained O2 sat at room air <=93% “
123
Q
"Most common cause of peritonitis in nephrotic syndrome
A. Staphylococcus aureus
B. Streptococcus pyogenes
C. Hemophilus influenza type B 
D. Streptococcus pneumoniae"
A

D

“Nelson 21st p2755
Children with nephrotic syndrome are at significantly increased risk for infection with encapsulated bacterial and, in particular, pneumococcal disease. Spontaneous bacterial peritonitis presents with fever, abdominal pain, and peritoneal signs. Although pneumococcus is the most frequent cause of peritonitis, gram negative bacteria also are associated with a signficant number of cases. “

124
Q

“What is the modality of choice to clearly define structures in the pelvis and the suprarenal area?
a. Plain KUB
b. MRI
C. CT- Scan”

A

C

125
Q

“Child with proteinuria if exposed to this drug can cause nephrotic syndrome?

a. Trimethadione
b. Streptomycin
c. Both of the above
d. None of the above”

A

A

“Nelson 21st p2760
Nephrotic syndrome has also developed during therapy with numerous drugs and chemicals:
1. Histologic features of membranous glomerulopathy - penicillamine, captopril, gold, NSAIDS, mercury compounds
2. Histologic features of MCNS - probenecid, ethosuximide, methimazole, lithium
3. Histologic features of proliferative glomerulonephritis - procainamide, chlorpropamide, phenytoin, trimethadione, paramethadione)”

126
Q

“What is the expected acid-base imbalance in Aspirin overdose

a. metabolic acidosis with high anion gap
b. metabolic acidosis with normal anion gap
c. metabolic alkalosis with high anion gap
d. metabolic alkalosis with normal anion”

A

A

“Nelson 21st p500
Classically, laboratory values from a patient poisoned with salicylates reveal a primary respiratory alkalosis and a primary elevated anion gap metabolic acidosis. Early in the course of acute salicylism, respiratory alkalosis dominates and the patient is alkalemic. As the respiratory stimulation diminishes, the patient will move toward acidemia. Hyperglycemia (early) and hypoglycemia (late) have been described. Abnormal coagulation studes and acute kidney injury may be seen but are not common. “

127
Q
"A 2-year old male boy with intractable vomiting was brought to the ER due to spasmodic contractions of the muscles of the wrists. What is the most likely cause?
A. Hypocalcemia
B. Hyponatremia
C. Hypochloremia
D. Hypokalemia"
A

A

128
Q

“UTI is present if the specimen taken via suprapubic aspiration has:
A. 105 CFU/ml growth of a single pathogen
B. > 1000 CFU/ml of a single pathogen
C. 50,000 CFU/ml of a single pathogen
D. Growth of any urinary pathogen in any number”

A

D

“Urine Culture Interpretation –

Suprapubic aspiration
Growth of urinary pathogen in any number (except 2-3 x 1000 CFU of coagulase-negative staphylococci)

Catheterization
Febrile infants or children usually having 50,000 CFU of a single urinary pathogen, but infection may be present with counts >1000 CFU

Midstream clean-void
Symptomatic patients usually having 10,000 CFU of a single urinary pathogen

Midstream clean-void
Asymptomatic patients having at least two specimens on different days with 10,000 of same pathogen.

*Routine reculturing of urine after 2 days of antibiotics generally not necessary.”

129
Q

“Which of the following statements is TRUE of hemolytic-uremic syndrome?
A. The findings on urinalysis consist of massive hematuria and heavy proteinuria.
B. The prognosis is poor in spite of aggressive treatment.
C. The onset is usually preceded by gastroenteritis.
D. The platelet count is markedly elevated.”

A

C

“Nelson 21st p2734
The diagnosis is made by the combination of microangiopathic hemolytic anemia with schistocytes, thrombocytopenia, and some degree of kidney involvement.

Thrombocytopenia is an invariable finding in the acute phase, with platelet counts usually 20,000 - 100,000/mm3.

Urinalysis typically shows microscopic hematuria and low grade proteinuria

With early recognition and intensive supportive care, the mortality rate for diarrhea-associated HUS is <5% in most major medical centers. Up to half of patients may require dialysis support during the acute phase of the illness. Recovery of platelet counts usually occurs first, followed by renal recovery about 5 days later, and finally by resolution of anemia. Most recover renal function completely, but of surviving patients, 5% remain dependent on dialysis, and up to 30% are left with some degree of chronic renal insufficiency. “

130
Q

“A 6-year old male had painless gross hematuria of 2 days, & periorbital edema & scanty urine on the day of consultation. He had a history of sore throat a week ago. The urinalysis showed tea-colored urine with pH 6, Sp Gr 1.025, no sugar, protein +1, RBC >100 / HPF, WBC 4 - 5 / HPF & RBC cast 0-1 / LPF. What is the most likely diagnosis?
A. Acute hemorrhagic cystitis
B. Urolithiasis
C. Benign familial hematuria
D. Acute post-infectious glomerulonephritis”

A

D

“Nelson 21st p2723-2724
APSGN is a classic example of the acute nephritic syndrome characterized by the sudden onset of gross hematuria, edema, hypertension, and renal dysfunction.

APSGN follows infection of the throat or skin by certain nephritogenic strains of group A beta hemolytic streptococci. Poststreptococcal GN commonly follows streptococcal pharyngitis during cold-weather months and streptococcal skin infections or pyoderma during warm-weather months.

PSGN is most common in children ages 5-12 yr and uncommon before the age of 3 yr. The typical patient develops an acute nephritic syndrome 1-2 wk after an antecedent streptococcal pharyngitis or 3-6 wk after a streptococcal pyoderma.

Urinalysis demonstrates RBC, often in association with blood cell casts, proteinuria, and polymorphonuclear leukocytes. “

131
Q

“True of SLE in children

a. Nephritis is more common in children
b. Faster progression to ESRD
c. Nonspecific findings are present at disease onset”

A

A

“Nelson 21st p2729-2730
Although SLE is less frequent in children, renal involvement (lupus nephritis) is more common and more severe than that seen in adults.

The clinical findings in patients having milder forms of lupus nephritis (all class I and II, some class III) have hematuria, normal renal function, and proteinuria <1g/24hr. Some patients with class III and all patients with class IV nephritis have hematuria and proteinuria, active urinary sediment with cellular casts, hypertension, reduced renal function, nephrotic syndrome, or acute kidney injury. Patients with class V nephritis commonly present with nephrotic syndrome.

Overall, renal survival (defined as chronic kidney disease without need for end-stage renal disease therapy) is seen in 80% of patients 10 yr after diagnosis of SLE nephritis. Patients with diffuse proliferative WHO class IV lupus nephritis, poor renal function at presentation, or persistent nephrotic-range proteinuria exhibit the highest risk for progression to end-stage renal disease.”

132
Q
"Best screening test for SLE
A. ANA
B. Anti-Smith
C. Anti-Ro
D. Anti- DsDna"
A

A

“Nelson 21st p1275.
Of note, although a positive ANA test result is not required for the diagnosis of SLE, ANA-negative lupus is extremely rare. ANA is very sensitive for SLE (95-99%) but is not very specific (50%).

Antibodies against dsDNA and anti-Smith are specific for SLE (98%) but not as sensitive (40-65%)

Nelson 21st p1277 Table 183.6 Autoantibodies typically associated with systemic lupus erythematosus

Anti-dsDNA

  • Specific for the diagnosis of SLE
  • Correlates with disease activity, especially nephritis

Anti-Ro antibody (anti-SSA antibody), anti-La antibody (anti-SSB antibody)

  • Associated with sicca syndrome
  • May suggest diagnosis of Sjogren syndrome
  • Increased risk of neonatal lupus in offspring (congenital heart block)
  • May be associated with cutaneous and pulmonary manifestations of SLE
  • May be associated with isolated discoid lupus “
133
Q

“Patient with rashes and arthritis in the extremities. What is least likely?
A. JIA
B. Ankylosing spondylitis
C. Lyme disease”

A

B

“Nelsons 21st p1269
Clinical manifestations that help distinguish spondyloarthritis from other forms of juvenile arthritis include arthritis of the axial skeleton (sacroiliac joints) and hips, enthesitis (inflammation at the site of tendon, ligament, or joint capsule attachment to bone), symptomatic eye inflammation (acute anterior uveitis), and GI inflammation (even in the absence of IBD)

Nelson 21st p1262
Systemic JIA is characterized by arthritis, fever, rash, and prominent visceral involvement, including hepatosplenomegaly, lymphadenopathy, and serositis (pericarditis)

Nelson 21st p1606. Table 249.1. Clinical stages of Lyme disease

  1. Early localized (3-30 days) - erythema migrans (single), variable constitutional symptoms (headache, fever, myalgia, arthralgia, fatigue)
  2. Early disseminated (3-12 wk) - erythema migrans (single or multiple), worse constitutional symptoms, cranial neuritis, meningitis, carditis, ocular disease
  3. Late (>2mo) - arthritis typically involving large joints, especially the knee”
134
Q
"True of scleroderma except
A. Muscle enzymes are diagnostic
B. Raynaud is most common initial symptom in pediatrics
C. Both of the above are true
D. None of the above are true"
A

A

”"”Scleroderma
Range of conditions unified by presence of skin fibrosis. Juvenile localized scleroderma (JLS/morphea) is more common. Juvenile systemic sclerosis has multiorgan involvement

Mechanism of disease is a combination of vasculopathy, autoimmunity, immune activation and fibrosis. Triggers injure vascular cells, which promotes fibrosis.

Vascular changes include Raynaud phenomenon and pulmonary hypertension. Raynaud phenomenon is the most frequent initial symptom - classic triphasic sequence of blanching, cyanosis, and erythema of the digits induced by cold exposure/emotional stress

No diagnostic test is conclusive for scleroderma. Most patients are ANA positive and antihistone positive.

Treatment for topical lesions involves topical corticosteroids or UV radiation. For deep lesions, methotrexate and systemic corticosteroids are used.

(Nelson 21st Chap 185)”””

135
Q

“12y/o child that complains of prolonged fever with evanescent rash, lymphadenopathy and joint pains on the right knee?

a. Septic arthritis
b. Juvenile Idiopathic arthritis
c. Juvenile Rheumatoid arthritis
d. SLE”

A

B

“Nelson 21st p1262
Systemic JIA is characterized by arthritis, fever, rash, and prominent visceral involvement, including hepatosplenomegaly, lymphadenopathy, and serositis (pericarditis)

The characteristic fever, defined as spikeing temperatures to >=39C, occurs on a daily or twice-daily basis for at least 2 wk, with a rapid return to normal or subnormal temperatures

The evanescent salmon-colored lesions, classic for sJIA, are linear or circular and are usually distributed over the trunk and proximal extremities. The classic rash is nonpruritic and migratory with lesions lasting <1 hr. Koebner phenomenon, a cutaneous hypersensitivity in which classic lesions are brought on by superficial trauma, is often present. Heat can also evoke rash. “

136
Q

“7 y/o child with fever, facial rash, chest pain and arthritis of right ankle and knee?

a. SLE
b. Juvenile dermatomyositis
c. Undifferentiated arthritis
d. HUS”

A

A

“Nelson 21st p1275
Any organ system can be involved in SLE, so the potential clinical manifestations are myriad. The most common presenting complaints of children with SLE include fever, fatigue, hematologic abnormalities, arthralgia, and arthritis. Arthritis is usually present in the 1st yr of diagnosis; arthritis may be painful or painless swelling, often with stiffness in the morning, and is usually a symmetric polyarthritis affecting large and small joints. “

137
Q

“A child with arthritis,rash and abdominal pain?

a. HUS
b. SLE
c. Juvenile dermatomyositis
d. Undifferentiated arthritis”

A

B

“Nelson 21st p1275
Any organ system can be involved in SLE, so the potential clinical manifestations are myriad. The most common presenting complaints of children with SLE include fever, fatigue, hematologic abnormalities, arthralgia, and arthritis. Arthritis is usually present in the 1st yr of diagnosis; arthritis may be painful or painless swelling, often with stiffness in the morning, and is usually a symmetric polyarthritis affecting large and small joints.

Nelson 21st p1275 Table 183.1 Potential clinical manifestations of SLE

GI - hepatosplenomegaly, pancreatitis, vasculitis affecting bowel, protein losing enteropathy, peritonitis “

138
Q

Complication of juvenile dermatomyositis.

A

Bowel perforation

“Nelson 21st p1284
Most complications from JDM are related to prolonged and severe weakness from muscle atrophy to cutaneous calfications and scarring or atrophy to lipodystrophy. Secondary complications from medical treatments are also common. Children with acute and severe weakness are at risk for aspiration pneumonia and respiratory failure and occasionally require NGT feeding and MV support until weakness improves.

Rarely, vasculitis of the GI tract develops in children with severe JDM. Crampy abdominal pain and occult GI bleeding may indicate bowel wall vasculitis and lead to ischemia, GI bleeding and perforation if nottreated with complete bowel rest and aggressive treatment for the underlying inflammation. “

139
Q

Most dreaded complication of Takayasu arteritis

A

Arterial aneurysmal rupture

“Nelson 21st p1321-1322
Progressive vascular damage can result in arterial stenoses, aneurysms, and occlusions, which produce ischemic symptoms and can be organ or life threatening. Potential ischemic complications include stroke, renal impairment or failure, myocardial infarction, mesenteric ischemia, and limb-threatening arterial disease.

140
Q

In kawasaki: at risk for development of coronary artery abnormality

A

Fever more than 10 days

141
Q

Treatment of Kawasaki

A

IVIG AND ASPIRIN

“Nelson 21st p1315
Patients with acute KD should be treated with 2g/kg of IVIG as single infusion usually administered over 10-12hr within 10 days of disease onset, and ideally as soon as possible after diagnosis.

In addition, moderate to high dose aspirin should be administered until the patient is afebrile, then lowered to antiplatelet doses. Aspirin i scontinued for its antithrombotic effect until 6-8 wk after illness onset and is then discontinued in patients who have had normal 2D echo findings thoghout the course of their illness.

Corticosteroids have been used as a primary therapy with the 1st dose of IVIG in the hopes of improving coronary outcomes. “

142
Q

“Mom of patient you diagnosed with HSP is worried about the renal complications she read on internet . What advise will you give?
A. Serial urinalysis should be done to check for microscopic hematuria
B. Renal biopsy should be done
C. Use of steroid will prevent of developing renal disease”

A

A

“Nelson 21st p2731
The severity of the systemic manifestations is not correlated with the severity of the nephritis. Most patient who develop nephritis have urinary abnormalities by 1mo, and nearly all have abnormalities by 3-6mo after the onset of HSP. Therefore, a urinalysis should be performed weekly in patients with HSP during the period of active clinical disease. Thereafter a urinalysis should be performed once a month for up to 6mo. If all urinalyes are normal during this follow-up interval, nephritis is unlikely to develop.

Indications of a kidney biopsy in children with HSP nephritis include significant proteinuria (urine protein >1g/day or urine protein/creatinine ratio >1.0), significant hypertension, or elevated serum creatinine. “

143
Q

“Sport not recommended for patient with uncontrolled seizure

a. Swimming
b. Table tennis
c. Billiards
d. Bowling”

A

A

“Nelson 21st p3682 Table 706.3 Medical conditions and sports participation

Seizure disorder, well-controlled - may participate in sports
- Risk of seizure during participation is minimal

Seizure disorder, poorly controlled - qualified yes for sports participation

  • Athlete needs individual assessment for collision, contact, or limited-contact sports
  • The following noncontact sports should be avoided - archery, riflery, swimming, weightlifting, power lifting, strength training, and sports involving heights; in these sports, a seizure during activity can pose a risk to self or others”
144
Q
"Sport not to be done by HIV positive patient?
a. Swimming
b. Boxing
C. Weightlifting
d. Gymnastics"
A

B

“Nelson 21st p3682 Table 706.3 Medical conditions and sports participation

HIV - may participate in sports

  • Because of the apparent minimal risk to others, all sports may be played as the althlete’s state of health allows (especially if viral load is undetectable or very low)
  • For all athletes, skin lesions must be covered properly, and athletic personnel should use universal precautions when handling blood or bodily fluids with visible blood
  • Certain sports (such as wrestling or boxing) can create a situation that favors viral transmission (likely bleeding plus skin breaks); if viral load is detectable, then athletes should be advised to avoid such high-contact sports”
145
Q

“Sport not to be done by asthmatic child?

a. Scuba diving
b. Bowling
c. Billiards
d. Table tennis”

A

A

“Nelson 21st p3682 Table 706.3 Medical conditions and sports participation

Asthma - may participate in sports

  • With proper medication and education, only athletes with severe asthma need to modify their participation
  • For those using inhalers, recommend having a written action plan and using a peak flowmeter daily
  • Athletes with asthma might encounter risks with scuba diving “
146
Q

“Contact sports not for obese because?

a. Risk of injury
b. Because of increased risk of heat illness and cardiovascular strain
c. Both of the above”

A

B

“Obesity - may participate in sports
- Because of the increased risk of heat illness and cardiovascular strain, obese athletes particularly need careful acclimatization (to the environment and to exercise intensity and duration), sufficient hydration, and potential activity and recovery medications during competition and training.”

147
Q

“Multi drug resistant TB is resistant t o:

a. lsoniazid & Rifampicin
b. lsoniazid & Ethambutol
c. Isoniazid & Pyrazinamide
d. Rifampicin & Pyrazinamide”

A

A

“TBIC 2016 p72-73
Classification based on Drug Susceptibility Testing
1. Monoresistant TB - resistance to one first line anti-TB drug only
2. Polydrug resistant TB - resistance to more than one first line anti-TB drug (other than both isoniazid and rifampicin)
3. Multi-drug resistant TB - resistance to at least both isoniazid and rifampicin
4. Extensively drug-resistant TB (XDR-TB) - resistance to any fluouroquinolone and at least one of the three second-line injectable drug (capreomycin, kanamycin, and amikacin) in addition to multidrug-resistance
5. Rifampicin-resistant TB (RR-TB) - resistance to rifampicin, detected using phenotypic or genotypic methods, with or without resistance to other anti-TB drugs. It includes any resistance to rifampicin, whether monoresistance, multi-drug resistance, poly-drug resistance or extensive drug resistance”

148
Q

“How many weeks from illness can TST be positive

a. 2-12 weeks
b. 8-16 weeks
c. 8-12 weeks
d. 2-16 weeks”

A

A

“TBIC 2016 p83
In most children, the delayed-type hypersensitivity response to tuberculin, which is the estimated interval between TB exposure and detectable skin test reactivity (referred to as the window period) ranges from 2-12 weeks. Analysis of data collected from studies indicates that 8 weeks is the outer limit of the window period. A negative test result obtained less than 8 weeks after exposure is therefore considered unreliable for excluding TB infection.”

149
Q

“Treatment of Category 1A TB

a. 2HRZE/4HR
b. 2HRZE/10HR
c. 2HRZES/1HRZE/5HRE
d. 2HRZES/1HRZE/9HRE”

A

A

"TBIC 2016 p151
Cat I - 2HRZE / 4HR
Cat Ia - 2HRZE / 10HR
Cat II - 2HRZES / 1HRZE / 5HRE
Cat IIa - 2HRZES / 1HRZE / 9HRE "
150
Q

“Xray findings in Primary TB EXCEPT

a. Calcifications
b. Lymphangitis
c. Pleural effusion”

A

A

“TBIC 2016 p95
In primary TB, the following radiographic changes may be seen: parenchymal involvement (primary focus), lymphangitis, localized pleural effusion, and regional lymphadenitis. The primary complex is composed of all the above 4 findings. The most common radiologic finding is lymphadenopathy “